ALL FAR

Pataasin ang iyong marka sa homework at exams ngayon gamit ang Quizwiz!

Goodwill = Cost - your portion of fair value of the investee's net assets

How do you calculate goodwill under the equity method of recording investments?

Answer (C) is correct. An entity may elect the fair value option (FVO) for most recognized financial assets and liabilities. The decision whether to elect the FVO is made irrevocably at the election date. The decision is made instrument by instrument and only for an entire instrument.

When valuing certain financial instruments, a company that has elected the fair value measurement option must apply the accounting measurement based on which of the following criteria? A. Type-by-type basis. B. At the entity level. C. Instrument-by-instrument basis. D. A portion of an asset or liability.

serial

A ________ bond matures in stated amounts at regular intervals.

Accumulated oci

What is oci closed into?

if they have a finite useful life

When may trademarks be amortized?

tier 1- proprietary funds tier 2- enterprise funds

where do funds from parking garages or utilities go?

governmental fund accounting

Current financial resources measurement focus and modified accrual accounting only pertain to

GP = net sales - COGS

Gross profit =

Expensed when occurred.

How are expenses related to debt restructuring accounted for?

LIFO is not an accepted accounting policy. Also, you must recognize an inventory loss from a market decline even if no loss is reasonably expected for the year.

IFRS difference for interim financial statements for inventory valuation

Answer (B) is correct. The term "realization" is used most precisely in accounting and financial reporting with regard to sales of assets for cash or claims to cash. The terms "realized" and "unrealized" identify revenues or gains and losses on assets sold and unsold, respectively. Thus, the sale of depreciated equipment results in realization.

According to the FASB's conceptual framework, which of the following statements conforms to the realization concept? A. Cash was collected on accounts receivable. B. Depreciated equipment was sold in exchange for a note receivable. C. Product unit costs were assigned to cost of goods sold when the units were sold. D. Equipment depreciation was assigned to a production department and then to product unit costs.

allowance for doubtful accounts. It's called net realizable value.

AR is reported as a current asset net of

current financial resources, specifically financial position and changes in financial position

The measurement focus of governmental funds financial statements is on

1. Enterprise funds 2. Internal service funds

What are the two proprietary funds called

Inventory Turnover = COGS / Avg inventory

inventory turnover =

improve

If a ratio is less than 1.0, a transaction that results in equal increases in the numerator and denominator will _______________ the ratio.

1. Extends the useful life of the asset 2. Increases the number of units produced 3. Increases the quality of units produced 4. Increases the efficiency of the asset Q quality E extends life E efficiency N number of units produced increases

If an improvement does any one of these four things, you capitalize it.

capitalize

If you make an improvement to a fixed asset, do you capitalize or expense it? When it's called a betterment- you ________ When it's called an addition- you _________ When it's an extraordinary repair- you _______

undiscounted future cash flows expected from the use and disposition of the asset.

Impairment: Recoverability test. The carrying amount is not recoverable if it exceeds the sum of the

Answer (B) is correct. Based on fair value, goodwill is the excess of (1) the sum of (a) the consideration transferred ($8,000,000), (b) any noncontrolling interest in the acquiree ($0), and (c) the acquirer's previously held equity interest in the acquiree ($0) over (2) the net of the identifiable assets acquired and liabilities assumed ($5,000,000). Thus, goodwill is $3,000,000, and the increase in net assets is $8,000,000. The acquisition-related cost (the finder's fee) is expensed.

In a business combination, Major Corporation issued nonvoting, nonconvertible preferred stock with a fair value of $8 million in exchange for all of the outstanding common stock of Minor Corporation. On the acquisition date, Minor had identifiable net assets with a carrying amount of $4 million and a fair value of $5 million. In addition, Major issued preferred stock with a fair value of $800,000 to an individual as a finder's fee in arranging the transaction. As a result of this transaction, Major should record an increase in net assets of A. $5,000,000 B. $8,000,000 C. $4,000,000 D. $8,800,000

Answer (C) is correct. According to ASC 350-20-20, goodwill is an asset representing the future economic benefits arising from other assets acquired in a business combination that are not individually identified and separately recognized. It is the excess of consideration transferred over the fair value of the net identifiable assets acquired.

In a business combination, the valuation of goodwill is a calculation A. Of all of the unlimited life intangible assets. B. Of all of the increases in market valuation of the intangible assets acquired. C. Of the residual paid above the fair value of the identifiable net assets. D. To offset the bargain purchase cost.

the net purchases

In a periodic inventory system, the beginning inventory is equal to the total goods available for sale minus

Answer (B) is correct. Investing activities include the lending of money; the collection of those loans; and the acquisition, sale, or other disposal of (1) loans and other securities that are not cash equivalents and that have not been acquired specifically for resale and (2) property, plant, equipment, and other productive assets.

In a statement of cash flows, receipts from sales of property, plant, and equipment and other productive assets should generally be classified as cash inflows from A. Selling activities. B. Investing activities. C. Financing activities. D. Operating activities.

LIFO

In a time of rising prices (inflation), use of the ________ method results in the lowest year-end inventory, the highest cost of goods sold, and the lowest gross profit.

Answer (A) is correct. In accordance with the if-converted method, the diluted earnings per share calculation assumes that dilutive convertible debt is converted into common stock at the beginning of the period or at the time of issuance, if later. Given the assumed conversion, no debt would exist upon which interest could have been paid. Interest is a deduction in arriving at net income. Accordingly, that interest savings, net of tax effect, should be added back to net income in the diluted earnings per share computation.

In determining earnings per share, interest expense, net of applicable income taxes, on dilutive convertible debt should be A. Added back to net income for diluted earnings per share. B. Added back to net income for basic earnings per share and ignored for diluted earnings per share. C. Deducted from net income for diluted earnings per share. D. Added back to net income for both basic and diluted earnings per share.

Answer (C) is correct. The units-of-production depreciation method allocates asset cost based on the level of production. As production varies, so will the credit to accumulated depreciation. Consequently, when an asset's service potential declines with use, the units-of-production method is the most appropriate method.

In which of the following situations is the units-of-production method of depreciation most appropriate? A. An asset is subject to rapid obsolescence. B. An asset incurs increasing repairs and maintenance with use. C. An asset's service potential declines with use. D. An asset's service potential declines with the passage of time.

A. IFRS will allow the restoration of an impairment loss, but U.S. GAAP will not allow the restoration.

Last year, Katt Co. reduced the carrying amount of its long-lived assets used in operations from $120,000 to $100,000, in connection with its annual impairment review. During the current year, Katt determined that the fair value of the same assets had increased to $130,000. What amount should Katt record as restoration of previously recognized impairment loss in the current year's financial statements? A.$0 B.$10,000 C.$20,000 D.$30,000

debt to finance assets and operations. debt to equity ratio = liabilities / SE

Leverage ratios measure the firm's use of

put; call

A _______ option is the right to sell an asset at a fixed price on a future date. A ______ option is the right to purchase an asset at a fixed price on a future date.

term

A ___________ bond has a single maturity date at the end of its term.

loss. less than the commitment price. Debit Unrealized holding loss -- earnings Credit Liability -- purchase commitment

A commitment to acquire goods in the future is not recorded at the time of the agreement, e.g., by debiting an asset and crediting a liability. But a __________ is recognized on a firm, noncancelable, and unhedged purchase commitment (unconditional purchase obligation) if the market price of the goods is _____________________________.

FIFO

A disadvantage of _________ is that current revenues are matched with older costs.

Answer (C) is correct. The amortization period for an intangible asset distinct from goodwill is the shorter of its useful life or the legal life remaining after acquisition.

A purchased patent has a remaining legal life of 15 years. It should be A. Expensed in the year of acquisition. B. Amortized over 40 years. C. Amortized over its useful life if less than 15 years. D. Amortized over 15 years regardless of its useful life.

LIFO

A significant advantage of the _______________ method is its matching of current revenues with the most recent product costs

service-type

A warranty that can be purchased separately by the customer is a ___________________ warranty

Answer (D) is correct. Costs incurred to fulfill a contract must be capitalized (recognized as an asset) only if the costs meet all of the following criteria: (1) The costs relate directly to a current or anticipated contract that is specifically identifiable, (2) the costs generate or enhance resources of the entity that will be used in satisfying performance obligations in the future, and (3) the costs are expected to be recovered.

According to ASC 606, which of the following is a criterion for capitalization of costs incurred to fulfill a contract with a customer? A. The costs would have been incurred whether or not the contract was obtained. B. The costs relate to any future contract. C. The costs must be related to a satisfied performance obligation. D. The costs are expected to be recovered.

Answer (B) is correct. The activities and assets of a business are capable of being managed to provide economic benefits (returns such as dividends, and lower costs). Processes are applied to inputs to generate outputs. Outputs are direct returns to investors and other participants. C.

According to GAAP, a business must A. Generate a return. B. Be capable of being managed to provide economic benefits. C. Have goodwill. D. Have inputs, outputs, and processes.

Answer (C) is correct. Revenues should be recognized when they are realized or realizable and earned. The most common time at which these two conditions are met is when the product or merchandise is delivered or services are rendered to customers. Rental revenue should be recognized evenly over the period of time during which the asset is being rented.

According to the FASB's conceptual framework, which of the following bases is the best indication that rental revenue should be recognized as realized and earned? A. The signing of the rental contract. B. The collection of cash. C. The passage of time. D. Production and delivery.

face amount times

Accrued interest receivable is always equal to the __________________ the nominal rate for the period of the accrual.

Answer (A) is correct. The contingent liability for damages resulting from a lawsuit is noncontractual. However, if it is more likely than not at the acquisition date that a noncontractual contingency will result in a liability or an asset, it will be measured at acquisition-date fair value. Moreover, the indemnification asset and the indemnified item are recognized at the same time and on the same basis. Thus, the indemnification asset also is recognized at acquisition-date fair value.

Acquirer and Acquiree are the combining entities in a business combination. As part of the bargain, Acquirer assumed a contingent liability based on a suit brought against Acquiree because of a defect in one of its products. However, the former owner of Acquiree has agreed to pay the amount of any damages in excess of $5,000,000. In the consolidated balance sheet issued on the acquisition date, the contingent liability is reported at acquisition-date fair value. Accordingly. A. An indemnification asset is recognized at acquisition-date fair value. B. No indemnification asset is recognized until the contingency is resolved. C. An exception to the customary accounting for a business combination applies. D. A valuation allowance is reported for the indemnification asset.

Answer (A) is correct. An accrued expense has been incurred but not paid. Thus, it should be charged (matched) against revenue in the current period and recorded as a liability.

An accrued expense can best be described as an amount A. Not paid and currently matched with earnings. B. Paid and not currently matched with earnings. C. Paid and currently matched with earnings. D. Not paid and not currently matched with earnings.

not amortized and must be tested for impairment. The carrying amount of an intangible asset with an indefinite useful life equals its historical cost minus impairment losses.

An intangible asset with a definite useful life is amortized, but one with an indefinite life is

anticipatory liability (a credit). It records the amount authorized to be spent by the government for the fiscal period

Appropriations is an

Avg days sales = 365 / inventory turnover

Average days sales

If the rights are exercised and stock is issued, the issuer will reflect the proceeds received as a credit to (an increase in) common (preferred) stock at par value, with any remainder credited to additional paid-in capital.

Blue Co. issued preferred stock with *detachable* common stock warrants at a price that exceeded both the par value and the fair value of the preferred stock. At the time the warrants are exercised, Blue's total equity is increased by the Cash Received upon Exercise ?? or Carrying Amount of the Warrants ??

premium

Bonds are sold at a ________________ when the stated interest rate exceeds the market interest rate.

COGM = beg - end - COGS

COGM =

COGS = COGM + beg - end

COGS =

beginning material inventory +purchases -returns +freight-in -ending WIP -ending finished goods =COGS

COGS for a manufacturer

beginning inventory +purchases -returns +freight-in -ending inventory = COGS

COGS for a retailer

A business combination is accounted for as an acquisition. Under the acquisition method, the entry recording the transaction is based on the fair values exchanged. FV of long-term debt and the PPE.

Company J acquired all of the outstanding common stock of Company K in exchange for cash. The consideration transferred exceeds the acquisition-date fair value of the net assets acquired. How should Company J determine the amounts to be reported for the plant and equipment and long-term debt acquired from Company K?

fair value

Derivatives can be classified as assets or liabilities and they are measured at

Answer (C) is correct. When supplies are received by or services are rendered to a governmental unit, a journal entry is made to debit expenditures and to credit vouchers payable. Also, the previously recorded encumbrance must be reversed by debiting encumbrances outstanding and crediting encumbrances. Because the original encumbrance entry is reversed in full, encumbrances outstanding must be debited for the estimated cost of $5,000.

Elm City issued a purchase order for supplies with an estimated cost of $5,000. When the supplies were received, the accompanying invoice indicated an actual price of $4,950. What amount should Elm debit (credit) to encumbrances outstanding in its general fund after the supplies and invoice were received? A. $50 B. $4,950 C. $5,000 D. $(50)

Answer (C) is correct. When one entity controls another, consolidated financial statements must be issued regardless of the percentage of ownership. Control is the direct or indirect ability to determine the direction of management and policies of the investee. This usually means one entity's direct or indirect ownership of more than 50% of the outstanding voting interests of another entity.

For purposes of consolidating financial interests, a majority voting interest is deemed to be A. Greater than 50% of the directly or indirectly owned outstanding voting shares and at least 50% of the directly or indirectly owned outstanding nonvoting shares of another entity. B. 50% of the directly or indirectly owned outstanding voting shares of another entity. C. Greater than 50% of the directly or indirectly owned outstanding voting shares of another entity. D. 50% of the directly or indirectly owned outstanding voting shares and at least 50% of the directly or indirectly owned outstanding nonvoting shares of another entity.

FASB

GASB or FASB? voluntary health and wellness organization such as the red cross.

shorter

General improvements to leased property should be capitalized as leasehold improvements and amortized in accordance with the straight-line method over the _________________ of their expected useful life or the lease term.

1. *Imposed nonexchange revenues*- revenues result from assessments imposed on nongovernmental entities (such as property taxes, fines, and forfeitures). 2. *Derived tax revenues*- revenues that result from assessments imposed on exchange transactions (such as sales and income taxes) 3. *Government-mandated nonexchange transactions*- when one government provides resources to a government at another level and requires that they be used for a specific purpose (such as federal grant money that state governments are required to spend on primary education). 4. *Voluntary nonexchange transactions*- revenues resulting from agreements entered into willingly, such as private donations to municipal museums and grants from charitable organizations to build inner-city recreational facilities.

Governmental Revnue: 1. Imposed nonexchange revenues. 2. Derived tax revenues. 3. Government-mandated nonexchange transactions. 4. Voluntary nonexchange transactions.

groups of dissimilar assets with varying useful lives

Group and Composite Depreciation apply straight-line accounting to a collection of assets depreciated as if they were a single asset. The composite method applies to

similar assets.

Group and Composite Depreciation apply straight-line accounting to a collection of assets depreciated as if they were a single asset. The group method applies to

Answer (A) is correct. Cost of goods sold equals beginning finished goods, plus cost of goods manufactured for a manufacturer or purchases for a retailer, minus ending finished goods. Overstated ending inventory therefore results in understated cost of goods sold, overstated net income, and overstated retained earnings in the period of the error. When these errors reverse in the following period, beginning inventory and cost of goods sold will be overstated, and net income will be understated. Retained earnings will be correct.

Holly Company's inventory is overstated at December 31 of this year. The result will be A. Understated income next year. B. Understated income this year. C. Understated retained earnings this year. D. Understated retained earnings next year.

Carrying value is less than the fair value Debit loss Credit asset

How do you know an asset is impaired?

The carrying value on the books is greater than the selling price minus any cost to sell Debit loss Credit asset

How do you know if a held for sale asset is impaired?

At fair value as a current asset. Report the unrealized gains or losses in other comprehensive income. For trading securities you report the gains and losses in net income

How do you record available for sale securities on the balance sheet?

They are measured at amortized cost

How do you record held-to maturity asset investments?

It is not reported as dividend income. It instead is debited to cash and credited to the investment (asset)

How do you report a cash dividend under the equity method?

Current liability. It is not netted with cash regardless if its an account at a bank where there is a positive balance account.

How should an account that is overdrawn be classified as on the balance sheet?

the payments occur 1 year sooner.

The present value of the annuity due is greater than the present value of the ordinary annuity because

Participating

________________ bonds share in excess earnings of the debtor.

just authorized and issued.

are treasury shares considered authorized, issued, or outstanding?

stock dividend = less than 20-25% CS stock split = more than 20-25% of CS

at what % of common stock does the SEC require a dividend to be a stock split? stock dividend?

CA/CL

current ratio

debt ratio = liabilities / assets

debt ratio

Future deductible amount X Future tax rate = deferred tax asset

deferred tax asset =

Future taxable amount X Future tax rate = Deferred tax liability

deferred tax liability =

a governmental unit records revenue from a grant/donation when they spend it. a NFP unit records it when it's received.

difference between recording governmental donation revenue and NFP donation revenue

special purpose fund- expendable (can spend the principle) permanent fund- nonexpendable ( cannot spend the principle)

difference between the governmental special purpose fund and permanent fund

the converted-if methods. assume the preferred was converted to common at the earliest possible point. BUT THEN THERE WOULDN'T BE A PREFERRED DIVIDEND SO DON'T BACK OUT THE PREFERRED DIVIDENDS FOR THE YEAR. then compared the BEPS to the DEPS. if it doesn't go down, ignore the preferred stock because it's anti-dilutive.

dilutive EPS: what method do you use to see if convertible preferred stock/bonds are dilutive?

The discount on bonds payable originates when bonds are issued for less than the bond's face or maturity amount. A contra liability account that reports the amount of unamortized discount associated with bonds that are outstanding

discount on bonds payable

GM = Sales - COGS COGS =

gross margin =

general fund- different from governmental general fund.

internal reporting- private NPF hospital's account where they keep the *unrestricted* revenue/assets/long-term debt/agency funds/board-designated assets (it's internally restricted)

it's measured proportionately. you take the cash amount and divide it by the total consideration. That % is then multiplied by the potential gain. Then you back into the amount you'll debit for the new asset.

nonmonetary exchange when boot (less than 25% of the total consideration) is given and the assets are being measured at carrying amount- how do you measure/recognize a gain?

you now have to record the exchange at fair value as if it was a monetary exchange.

nonmonetary exchange when boot (more than 25% of the total consideration) is given and the assets are being measured at carrying amount- how do you measure/recognize a gain?

recognize the gain or loss. treat the boot as an extension of FV

nonmonetary exchange when boot is given and the assets are being measured at fair value-

if you can measure the FV, yes, you recognize the entire gain or loss. it's basically treated as a monetary exchange

nonmonetary exchange- if you can measure the fair value of the asset, do you recognize a gain or loss?

1. statement of operations 2. statement of changes in net assets- unrestricted, temporary restricted, and permanently restricted

nonprofit hospitals turn the statement of activities into two statements:

stock split

the primary purpose of a _______________________ is to improve the stock's marketability by reducing its market price and proportionally increasing the number of shares outstanding

stock dividend

the primary purpose of a ___________________________ is to provide shareholders with additional evidence of their interest in the RE of the business without distribution of cash or other assets.

debt service fund. it accumulated money to make interest payments and principal payments for long-term debt of the other four governmental funds

there is a fifth governmental fund called the

1. pension trust funds 2. investment trust funds 3. private-purpose trust funds 4. agency funds

tier 1: fiduciary funds- what falls under this fund

1. general fund- day-to-day/general operations 2. special revenue fund- revenue that is restricted for a special purpose 3. capital projects fund- construction of major assets 4. the permanent fund- restricted funds that are nonexpendable (you can never spend the original principle, only the dividends and interest) bonus: debt service fund

tier 1: governmental funds

1. enterprise funds 2. internal service funds

tier 1: proprietary funds- what falls under this fund

1. direct method- theoretically preferred (just a presentation difference, operating activities, gets presented differently) 2. indirect method- more heavily tested. solve most questions under the indirect method unless they tell you to use the direct method.

two methods for preparing the statement of cash flows:

public utility like gas, water, parking garage and electric. PUBLIC

under tier 2, what is an enterprise fund?

a government agency set up to service other governmental agencies.

under tier 2, what is an internal service fund?

when the government has a surplus of funds, they put it in here to invest it

under tier two, what is an investment trust fund?

perpetual; periodic

we use a moving average method to value inventory under a ____________________ system of accounting while we use just a weighted average method under the _________________ system of accounting

CR Ect. other finance uses

what account do you hit when a budget has on it that it will *give* an amount of money from another account?

DR Est. other finance sources

what account do you hit when a budget has on it that it will *receive* an amount of money from another account?

budgetary fund balance

what account do you hit when you need to balance a budget entry?

Answer (B) is correct. The parent's investment in subsidiary, intraentity dividends, and the subsidiary's equity accounts, which include retained earnings, are among the eliminations in a consolidation. The equity (net assets) of the subsidiary not directly or indirectly attributable to the parent is reported separately in consolidated equity as the noncontrolling interest. Consolidated retained earnings equals the accumulated earnings of the consolidated group not distributed to the owners of, or capitalized by, the parent. Thus, it equals the parent's retained earnings. Accordingly, the subsidiary's cash dividend reduces retained earnings reported in the subsidiary-only statements and the noncontrolling interest reported in the consolidated statements. But it does not affect consolidated retained earnings.

A 70%-owned subsidiary declares and pays a cash dividend. What effect does the dividend have on the retained earnings and noncontrolling interest balances in the consolidated balance sheet? A. No effect on either retained earnings or the noncontrolling interest. B. No effect on retained earnings and a decrease in the noncontrolling interest. C. Decreases in both retained earnings and the noncontrolling interest. D. A decrease in retained earnings and no effect on the noncontrolling interest.

patent

A ________________ is an intangible asset with a finite useful life, so it is subject to the recoverability test when testing for impairment.

finders fees affect net income. Registration fees are a DR to APIC. direct issuance costs of equity (underwriting, legal, accounting, tax, registration, etc.) are debited to additional paid-in capital.

A business combination must be accounted for as an acquisition. Which of the following expenses related to the business combination should be included, in total, in the determination of net income of the combined entity for the period in which the expenses are incurred? -Finders Fees -Registration fees for equity issued

-YES- Fees of finders and consultants -NO- Registration fees for equity securities issued Acquisition-related costs, such as finder's fees, professional and consulting fees, and general administrative costs, are expensed as incurred. However, direct issuance costs of equity (underwriting, legal, accounting, tax, registration, etc.) are debited to additional paid-in capital.

A business combination must be accounted for as an acquisition. Which of the following expenses related to the business combination should be included, in total, in the determination of net income of the combined entity for the period in which the expenses are incurred? -Fees of finders and consultants -Registration fees for equity securities issued

Answer (B) is correct. Net position has three components. Net investment in capital assets includes unrestricted and restricted capital assets, net of (1) accumulated depreciation and (2) related liabilities and deferred inflows and outflows of resources. Restricted net position includes restricted assets minus related liabilities and deferred inflows of resources. The restrictions are imposed by external entities (creditors, grantors, or other governments) or by law (constitutional provisions or enabling legislation). Unrestricted net position is the net of (1) assets, (2) deferred inflows and outflows of resources, and (3) liabilities not included in the other components of net position. Thus, it is a residual category. *Unrestricted net position includes items that may be internally committed or assigned (designated).* These commitments and assignments are not reported on the face of the statements.

A city council designates funds in the enterprise fund for future equipment replacement. The enterprise fund should report this as A. A restricted component of net position. B. An unrestricted component of net position. C. A designated component of net position. D. A net investment in capital assets.

percentages of a common base. The items in a balance sheet are usually stated in percentages of total assets. The items in the income statement are usually expressed as a percentage of sales. Thus, comparisons among firms in the same industry are possible despite differences in size. Comparison of firms in different industries has drawbacks because the optimal mix of assets, liabilities, etc., will vary from industry to industry.

A common-size financial statement presents the items in a financial statement as

Answer (D) is correct. In Year 2, the revaluation increase for the land is $15,000 ($105,000 fair value - $90,000 carrying amount). A revaluation increase must be recognized in other comprehensive income and accumulated in equity as a revaluation surplus. However, the increase must be recognized in profit or loss to the extent it reverses a decrease of the same asset that was recognized in profit or loss. In Year 1, the carrying amount of the asset was reduced by $10,000 ($100,000 - $90,000). This reduction was recognized in profit or loss (as there was no credit in revaluation surplus for the asset at that time). Thus, $10,000 of the increase in Year 2 must be recognized in profit or loss. The remaining $5,000 ($15,000 - $10,000) of the increase is recognized in other comprehensive income as a revaluation surplus.

A company has a parcel of land to be used for a future production facility. The company applies the revaluation model under IFRS to this class of assets. In Year 1, the company acquired the land for $100,000. At the end of Year 1, the carrying amount was reduced to $90,000, which represented the fair value at that date. At the end of Year 2, the land was revalued, and the fair value increased to $105,000. How should the company account for the Year 2 change in fair value? A. By recognizing $15,000 in other comprehensive income. B. By recognizing $10,000 in other comprehensive income. C. By recognizing $15,000 in profit or loss. D. By recognizing $10,000 in profit or loss and $5,000 in other comprehensive income.

since it is without consideration you only do a memorandum entyr.

A company issued rights to its existing shareholders. The rights were issued without consideration. how does the issuer record this?

Answer (B) is correct. Potential impairment of goodwill is deemed to exist only if the carrying amount of a reporting unit is greater than its fair value. The accounting for goodwill is based on the units of the combined entity into which the acquiree was absorbed. The goodwill impairment test includes an optional qualitative test and a two-step quantitative test.

A company reported $6 million of goodwill in last year's statement of financial position. How should the company account for the reported goodwill in the current year? A. Determine whether the fair value of the reporting unit is greater than the carrying amount and report the recovery of any previous impairment in the income statement. B. Determine whether the fair value of the reporting unit is less than the carrying amount and report an impairment loss on goodwill in the income statement. C. Determine whether the fair value of the reporting unit is greater than the carrying amount and report a gain on goodwill in the income statement. D. Determine the current year's amortizable amount and report the current year's amortization expense.

operating cycle = days' sales in receivables + says' sales in inventory

A firm's operating cycle is the amount of time that passes between the acquisition of inventory and the collection of cash on the sale of that inventory.

Answer (B) is correct. A material event or transaction that is unusual in nature, infrequent in occurrence, or both must be reported as a separate component of income from continuing operations. Such items must not be reported net of income taxes. The order of presentation in the income statement is Income from continuing operations Discontinued operations

A gain or loss from a transaction that is unusual in nature or infrequent in occurrence should be reported separately as a component of income A. After income from continuing operations and before results of discontinued operations. B. Before results of discontinued operations. C. Before cumulative effect of accounting changes and after results of discontinued operations. D. After results of income from continuing operations.

Answer (D) is correct. The proprietary funds financial statements are the statements of (1) net position; (2) revenues, expenses, and changes in fund net position; and (3) cash flows. A statement of cash flows also is required for entities engaged in business-type activities, e.g., public colleges and universities.

A government entity is required to include a statement of cash flows in which of the following financial statements? A. Governmental fund financial statements. B. Government-wide financial statements. C. Fiduciary fund financial statements. D. Proprietary fund financial statements.

Answer (C) is correct. An appropriation is an anticipatory liability (a credit). It records the amount authorized to be spent by the government for the fiscal period. A government commits to expend resources when a contract is signed or a purchase order is approved. The amount (an estimate of actual cost) then may be formally recorded in a budgetary account called an encumbrance. Encumbrance accounting may be used only for internal purposes in governmental funds, especially general and special revenue funds. The amount of available appropriations is $3,000 ($10,000 appropriated - $2,000 encumbrances - $5,000 expenditures). D.

A government's police department reports appropriations of $10,000, encumbrances of $2,000, and expenditures of $5,000. What is the amount of available appropriations for the police department? A. $7,000 B. $8,000 C. $3,000 D. $5,000

Answer (C) is correct. Transfers to and from accounts properly designated as appropriated retained earnings (such as general purpose contingency reserves or provisions for replacement costs of fixed assets) are always excluded from the determination of net income. However, appropriation of retained earnings is permitted if it is displayed within the equity section and is clearly identified. The effect of the appropriation is to restrict the amount of retained earnings available for dividends, not to set aside assets.

A retained earnings appropriation can be used to A. Smooth periodic income. B. Provide for a contingent loss that is probable and reasonable. C. Restrict earnings available for dividends. D. Absorb a fire loss when a company is self-insured.

Answer (D) is correct. General long-term liabilities are not directly related to and expected to be paid from proprietary and fiduciary funds. Thus, payment requires expenditure of governmental fund resources. Governmental funds are accounted for using the modified accrual basis, so expenditures for principal and interest on general long-term liabilities are generally recognized only when those amounts are due. Interest becoming due on the first day of the fiscal year (July 1) would not have been accrued in the prior year. The debt service process usually involves an interfund transfer (another financing use) from the general fund to the debt service fund. The debt service fund debits an expenditure (not an expense) on the legal due date. The recognition of the interfund transfer in the general fund and the expenditure in the debt service fund decreases the fund balance of each fund. Debt service also usually involves transfers of amounts to a fiscal agent, which makes the actual payments and performs other administrative tasks. Furthermore, a government may dispense with debt service funds if they are not (1) legally mandated or (2) used to accumulate resources for debt service in future years.

A state had general obligation bonds outstanding that required payment of interest on July 1 and January 1 of each year. State law allowed for the general fund to make debt payments without the use of a fiscal agent. The fiscal year ended June 30. Which of the following accounts would have decreased when the state paid the interest due on July 1? A. Interest expense. B. Interest expenditures. C. Interest payable. D. Fund balance.

Answer (D) is correct. A statement of changes in fiduciary net position reports (1) additions to, (2) deductions from, and (3) the net increase (decrease) for the year in net position for each fiduciary fund type. But an agency fund is not reported in the statement of changes in fiduciary net position. An agency fund does not have a net position. Its assets (plus deferred outflows of resources) equal its liabilities (plus deferred inflows of resources).

A state or local government's statement of changes in fiduciary net position A. Reports changes in net position for permanent funds. B. Reports changes in assets held in trust for other funds of the reporting entity. C. Includes changes that also are reported in the government-wide financial statements. D. Does not present information about changes in the net position of an agency fund.

Answer (D) is correct. A statement of cash flows is required only for proprietary funds and entities engaged in business-type activities (e.g., utilities, healthcare providers, and colleges and universities). Other fund types account for governmental activities, not business-type activities.

A statement of cash flows must be presented by a state or local government for which funds? A. All funds reported in the government-wide financial statements. B. Governmental funds. C. Fiduciary funds. D. Proprietary funds.

AR turnover = sales/avg AR

AR turnover

Answer (B) is correct. In a sale-leaseback transaction, if the lease qualifies as a capital lease, the gain on the sale is normally deferred and amortized by the seller-lessee in proportion to the amortization of the leased asset, that is, at the same rate at which the leased asset is depreciated. Thus, the deferred gain may be reported as an asset valuation allowance (a contra asset with a credit balance).

Able sold its headquarters building at a gain and simultaneously leased back the building. The lease was reported as a capital lease. At the time of sale, the gain should be reported as A. Item of other comprehensive income, net of income tax. B. An asset valuation allowance. C. Operating income. D. A separate component of equity.

Answer (B) is correct. The activities and assets of a business are capable of being managed to provide economic benefits (returns such as dividends, and lower costs). Processes are applied to inputs to generate outputs. Outputs are direct returns to investors and other participants.

According to GAAP, a business must A. Have inputs, outputs, and processes. B. Be capable of being managed to provide economic benefits. C. Have goodwill. D. Generate a return.

Answer (B) is correct. When (1) the net of the acquisition-date fair values of the identifiable assets acquired and liabilities assumed exceeds (2) the sum of the acquisition-date fair values of (a) the consideration transferred, (b) any noncontrolling interest in the acquiree, and (c) the acquirer's previously held equity interest in the acquiree, the acquirer recognizes the excess as a gain from bargain purchase.

Acquirer Corporation acquired for cash at $10 per share 100,000 shares of the outstanding common stock of Acquiree Company. The total fair value of the identifiable assets acquired minus liabilities assumed of Acquiree was $1.4 million on the acquisition date, including the fair value of its property, plant, and equipment (its only noncurrent asset) of $250,000. The consolidated financial statements of Acquirer Corporation and its wholly owned subsidiary must reflect A. Goodwill of $150,000. B. A gain of $400,000. C. A gain of $150,000. D. A deferred credit of $150,000.

Answer (B) is correct. The contingent liability for damages resulting from a lawsuit is noncontractual. However, if it is more likely than not at the acquisition date that a noncontractual contingency will result in a liability or an asset, it will be measured at acquisition-date fair value. Moreover, the indemnification asset and the indemnified item are recognized at the same time and on the same basis. Thus, the indemnification asset also is recognized at acquisition-date fair value.

Acquirer and Acquiree are the combining entities in a business combination. As part of the bargain, Acquirer assumed a contingent liability based on a suit brought against Acquiree because of a defect in one of its products. However, the former owner of Acquiree has agreed to pay the amount of any damages in excess of $5,000,000. In the consolidated balance sheet issued on the acquisition date, the contingent liability is reported at acquisition-date fair value. Accordingly. A. No indemnification asset is recognized until the contingency is resolved. B. An indemnification asset is recognized at acquisition-date fair value. C. An exception to the customary accounting for a business combination applies. D. A valuation allowance is reported for the indemnification asset.

unamortized cost or net realizable value.

After technological feasibility has been established for a software product, all software production costs incurred until the product is available for general release to customers shall be capitalized and subsequently reported at the lower of

Answer (B) is correct. The recipient of a stock dividend should not recognize income. After receipt of the dividend, the shareholder has the same proportionate interest in the corporation and the same total book value as before the declaration of the stock dividend.

Albert Co. acquired 4,000 shares of Nolan, Inc., common stock on October 20, Year 2, for $66,000. On November 30, Year 4, Nolan distributed a 10% common stock dividend when the market price of the stock was $25 per share. On December 20, Year 4, Albert sold 400 shares of its Nolan stock for $10,600. For the year ended December 31, Year 4, how much should Albert report as dividend revenue? A. $4,600 B. $0 C. $10,000 D. $10,600

Answer (D) is correct. Research and development's cost of internally developed patents must be expensed when incurred. Only relatively minor related costs, such as patent registration fees and legal fees, can be capitalized. Subsequent to the grant of a patent, its owner may need to defend a suit for patent infringement. The unrecovered costs of successful litigation are capitalized because they will benefit future periods. Thus, the amount that should be capitalized is $90,000 ($40,000 + $50,000).

Alta Co. spent $400,000 during the current year developing a new idea for a product that was patented during the year. The legal cost of applying for a patent license was $40,000. Also, $50,000 was spent to successfully defend the rights of the patent against a competitor. The patent has a life of 20 years. What amount should Alta capitalize related to the patent? A. $490,000 B. $40,000 C. $50,000 D. $90,000

FIFO

An advantage of _________ is that ending inventory approximates the market value.

events or changes in circumstances indicate that its carrying amount may not be recoverable.

An amortized intangible asset is reviewed for impairment when

deferred cost; asset

An asset provides future economic benefits. If a cash payment is made in one period and the recognition of the related expense (receipt of the benefit) is not appropriate until a later period, the ___________________ is recorded as an ____________.

Answer (D) is correct. When a reporting unit is disposed of in its entirety, goodwill of that reporting unit (to the extent an impairment loss has not been recognized) is included in the carrying amount of the reporting unit to determine the gain or loss on disposal. Consequently, the unimpaired goodwill of each reporting unit of the acquired entity is included in the total carrying amount of that entity.

An entire acquired entity is sold. The goodwill remaining from the acquisition should be A. Expensed in the period sold. B. Charged to retained earnings of prior periods. C. Charged to retained earnings of the current period. D. Included in the carrying amount of the net assets sold.

Answer (D) is correct. When a reporting unit is disposed of in its entirety, goodwill of that reporting unit (to the extent an impairment loss has not been recognized) is included in the carrying amount of the reporting unit to determine the gain or loss on disposal. Consequently, the unimpaired goodwill of each reporting unit of the acquired entity is included in the total carrying amount of that entity.

An entire acquired entity is sold. The goodwill remaining from the acquisition should be A. Expensed in the period sold. B. Charged to retained earnings of the current period. C. Charged to retained earnings of prior periods. D. Included in the carrying amount of the net assets sold.

D. The freight cost should be capitalized as part of property, plant, and equipment, and the interest cost should be immediately expensed. The costs to buy equipment, along with the costs to bring it to its location for use and make it ready for use, are capitalized into the cost of the equipment. Any interest costs in financing the purchase of equipment (which is otherwise ready to use) are finance (interest) costs and are expensed.

An entity purchased new machinery from a supplier before the entity's year-end. The entity paid freight charges for the purchased machinery. The entity took out a loan from a bank to finance the purchase. Under IFRS, what is the proper accounting treatment for the freight and interest costs related to the machinery purchase? A. The freight and interest costs should be immediately expensed. B. The freight and interest costs should be capitalized as part of property, plant, and equipment. C. The interest cost should be capitalized as part of property, plant, and equipment, and the freight cost should be immediately expensed. D. The freight cost should be capitalized as part of property, plant, and equipment, and the interest cost should be immediately expensed.

Answer (A) is correct. Interfund transfers are one-way asset flows with no repayment required. In a governmental fund, a transfer is an other financing use (source) in the transferor (transferee) fund. In a proprietary fund's statement of revenues, expenses, and changes in fund net position, transfers should be reported separately after nonoperating revenues and expenses. This component includes (1) capital contributions, (2) additions to endowments, and (3) special and extraordinary items.

An interfund transfer A. Is reported in a proprietary fund's statement of revenues, expenses, and changes in fund net position after nonoperating revenues and expenses. B. Is reported in a proprietary fund as an other financing source or use. C. Results in a receivable and a payable. D. Is the internal counterpart to an exchange or an exchange-like transaction.

Answer (D) is correct. In accounting for a stock dividend, the fair value of the additional shares issued is reclassified from retained earnings to capital stock and the difference to additional paid in capital. Stock dividend distributable is an item of shareholders' equity and not a liability.

An undistributed stock dividend declared by the Board of Directors should be reported as a(n) A. Long-term liability. B. Footnote to the financial statements. C. Current liability. D. Item in the shareholders' equity section.

Total capitalized cost times the revenue ratio (Annual gross software revenue ÷ Total projected gross revenue) Straight-line amortization- not taking out the CY (Total capitalized cost ÷ Estimated economic life of the software). To get the carrying amount for this it's the greater of NRV or the amount abover

Annual Amortization of Capitalized Software Costs is the greater of

Answer (B) is correct. The FASB's Accounting Standards Codification and SEC pronouncements are the only sources of authoritative financial accounting guidance for nongovernmental entities in the U.S. All other sources of guidance are nonauthoritative.

Arpco, Inc., a for-profit provider of healthcare services, recently purchased two smaller companies and is researching accounting issues arising from the two business combinations. Which of the following accounting pronouncements are the most authoritative? A. FASB Accounting Standards Updates. B. The Accounting Standards Codification. C. FASB Statements of Financial Accounting Concepts. D. FASB Statements of Financial Accounting Standards.

the lower of carrying amount or fair value minus cost to sell. A loss is recognized for a write-down to fair value minus cost to sell. A gain is recognized for any subsequent increase but only to the extent of previously recognized losses for write-downs.

Assets held for sale are measured at

Answer (C) is correct. In a consolidated balance sheet, reciprocal balances, such as receivables and payables, between a parent and a consolidated subsidiary are eliminated in their entirety, regardless of the portion of the subsidiary's stock held by the parent. Accordingly, the $300,000 receivable from Winn is eliminated. Because Grey cannot exercise significant influence over Carr, this investment should be accounted for on the fair-value basis. Receivables from an investee over which significant influence cannot be exercised are reported on the consolidated balance sheet. Grey should therefore report $200,000 in accounts receivable from affiliates.

At December 31, Grey, Inc., owned 90% of Winn Corp., a consolidated subsidiary, and 20% of Carr Corp., an investee over which Grey cannot exercise significant influence. On the same date, Grey had receivables of $300,000 from Winn and $200,000 from Carr. In its December 31 consolidated balance sheet, Grey should report accounts receivable from affiliates of A. $500,000 B. $230,000 C. $200,000 D. $340,000

Answer (A) is correct. In a business combination, acquisition-related costs, such as finder's fees, professional and consulting fees, and general administrative costs, are expensed as incurred. If this transaction were an acquisition of a group of assets (and not a business combination), the direct acquisition costs would be allocated on a pro rata basis to the assets acquired.

Bale Co. incurred $100,000 of acquisition costs related to the purchase of the net assets of Dixon Co. The $100,000 should be A. Expensed as incurred in the current period. B. Capitalized as part of goodwill and tested annually for impairment. C. Allocated on a pro rata basis to the nonmonetary assets acquired. D. Capitalized as an other asset and amortized over 5 years.

both. The objective is to provide information about the different types of business activities of the entity and the economic environments in which it operates. Disclosures include a measure of profit or loss and total assets for each reportable segment. Other items typically disclosed include revenues from external customers and other operating segments, interest revenue and expense, depreciation, depletion, amortization, unusual items, equity in the net income of equity-based investees, income tax expense or benefit, and other significant noncash items.

Bean Co. included interest expense and depreciation expense in its determination of segment profit, which Bean's chief financial officer considered in determining the segment's operating budget. Bean is required to report the segment's financial data in accordance with GAAP. Which of the following items should Bean disclose in reporting segment data? -Interest expense -Depreciation expense

Answer (D) is correct. These temporary differences arise from use of an accelerated depreciation method for tax purposes. Future taxable amounts reflecting the difference between the tax basis and the reported amount of the asset will result when the reported amount is recovered. Accordingly, Jab must recognize a deferred tax liability to record the tax consequences of these temporary differences. Deferred taxes are classified as noncurrent amounts.

Because Jab Co. uses different methods to depreciate equipment for financial statement and income tax purposes, Jab has temporary differences that will reverse during the next year and add to taxable income. Deferred income taxes that are based on these temporary differences should be classified in Jab's balance sheet as a A. Contra account to noncurrent assets. B. Current liability. C. Contra account to current assets. D. Noncurrent liability.

discount

Bonds are sold at a ______________ when the stated interest rate is less than the market interest rate.

Answer (A) is correct. Governmental reporting and reporting standards require that proprietary funds (enterprise and internal service funds) account for their operations on the accrual basis. Capital assets of proprietary funds should be reported in the government-wide statement of net position and the proprietary fund statement of net position. Depreciation expense is reported in the government-wide statement of activities and the proprietary fund statement of revenues, expenses, and changes in fund net position.

Capital assets of an enterprise fund should be reported in the A. Proprietary fund statement of net position, and depreciation on the capital assets should be recognized. B. Proprietary fund statement of net position, but no depreciation on the capital assets should be recognized. C. General fixed assets account group, but no depreciation on the capital assets should be recognized. D. General fixed assets account group, and depreciation on the capital assets should be recognized.

the lower of unamortized cost or net realizable value (NRV).

Capitalized software costs are reported at

B. $40,000 The interest cost to be capitalized is equal to the *lesser* of (a) the avoidable interest (based on the weighted-average amount of accumulated expenditures), or (b) the actual interest cost incurred. avoidable interest = $40,000. actual interest = ($50,000 + $20,000)

Cole Co. began constructing a building for its own use in January 20X1. During 20X1, Cole incurred interest of $50,000 on specific construction debt, and $20,000 on other borrowings. Interest computed on the weighted-average amount of accumulated expenditures for the building during 20X1 was $40,000. What amount of interest cost should Cole capitalize? A.$20,000 B.$40,000 C.$50,000 D.$70,000

Answer (A) is correct. Consolidated financial reporting is required when one entity owns, directly or indirectly, more than 50% of the outstanding voting interests of another entity. However, a majority-owned subsidiary is not consolidated if control does not rest with the majority owner.

Consolidated financial statements are typically prepared when one entity has a majority voting interest in another unless A. Control does not rest with the majority owner(s). B. The two entities are in unrelated industries, such as manufacturing and real estate. C. The fiscal year ends of the two entities are more than 3 months apart. D. The subsidiary is a finance entity.

capitalized as inventory.

Costs of producing computer software for sale (duplication of software, training materials, packaging) are

(1) cash and cash equivalents; (2) certain individual trading, available-for-sale, and held-to-maturity debt securities; (3) net receivables; (4) inventories; (5) prepaid expenses; and (6) certain individual investments in equity securities

Current assets include what 6 things

net realizable value; the net present value of the future cash flows

Current receivables are measured at ______________________________, and noncurrent receivables are measured at ______________________________.

Answer (C) is correct. Any error related to a prior period discovered after the statements are used must be reported as an error correction by restating the prior-period statements. If comparative statements are presented, corresponding adjustments must be made to net income (and its components) and retained earnings (and other affected balances) for all periods reported. As such, the cumulative effect of the error on Years 1 and 2 will be reflected in the carrying amounts of assets and liabilities as of the beginning of Year 3.

Cuthbert Industrials, Inc., prepares 3-year comparative financial statements. In Year 3, Cuthbert discovered an error in the previously issued financial statements for Year 1. The error affects the financial statements that were issued in Years 1 and 2. How should the company report the error? A. The financial statements for Years 1 and 2 should be restated; an offsetting adjustment to the cumulative effect of the error should be made to the comprehensive income in the Year 3 financial statements. B. The financial statements for Years 1 and 2 should not be restated; financial statements for Year 3 should disclose the fact that the error was made in prior years. C. The financial statements for Years 1 and 2 should be restated; the cumulative effect of the error on Years 1 and 2 should be reflected in the carrying amounts of assets and liabilities as of the beginning of Year 3. D. The financial statements for Years 1 and 2 should not be restated; the cumulative effect of the error on Years 1 and 2 should be reflected in the carrying amounts of assets and liabilities as of the beginning of Year 3.

C. The net of the acquisition-date fair value of the identifiable assets acquired and the liabilities assumed was $280,000 ($350,000 FV of assets - $70,000 FV of liabilities). The acquisition-date fair value of the consideration transferred was $210,000 [(20,000 shares × $10 FV per share) + $10,000 FV of contingent consideration]. Contingent consideration given in exchange for the acquiree is usually an obligation to transfer something to the former owners if a specified condition is met. Because the acquirer received 100% of the voting interests of the acquiree, no controlling interest or previously held equity interest in the acquiree existed. Consequently, the acquiree recognizes an ordinary gain in earnings of $70,000 ($280,000 FV net identifiable assets - $210,000 FV consideration transferred).

Damon Co. purchased 100% of the outstanding common stock of Smith Co. in an acquisition by issuing 20,000 shares of its $1 par common stock that had a fair value of $10 per share and providing contingent consideration that had a fair value of $10,000 on the acquisition date. Damon also incurred $15,000 in direct acquisition costs. On the acquisition date, Smith had assets with a book value of $200,000, a fair value of $350,000, and related liabilities with a book and fair value of $70,000. What amount of gain should Damon report related to this transaction? A. $250,000 B. $55,000 C. $70,000 D. $80,000

fair value

Determining impairment for an intangible asset: In a quantitative test, the _________________ of an asset is compared with its ________________.

Answer (B) is correct. Gain contingencies should not be recognized until they are realized. A gain contingency should be disclosed, but care should be taken to avoid misleading implications as to the likelihood of realization. Consequently, the only litigation award to be recognized in income in the current year is the $15,000 amount that has not been appealed. The other awards have not been realized because they have been appealed.

During January, Haze Corp. won a litigation award for $15,000 that was tripled to $45,000 to include punitive damages. The defendant, who is financially stable, has appealed only the $30,000 punitive damages. Haze was awarded $50,000 in an unrelated suit it filed, which is being appealed by the defendant. Counsel is unable to estimate the outcome of these appeals. In its current year financial statements, Haze should report what amount of pretax gain? A. $95,000 B. $15,000 C. $50,000 D. $45,000

Answer (B) is correct. The effects of intraentity transactions should be eliminated from consolidated financial statements in their entirety regardless of the parent's percentage of ownership. Consequently, consolidated selling expense is $1,450,000 ($1,100,000 + $400,000 - $50,000 of freight-out incurred on a sale by Pard to Seed). Seed's inventory balance is not relevant to this calculation because selling expenses, including freight-out, are not inventoried. C.

During the current year, Pard Corp. sold goods to its 80%-owned subsidiary, Seed Corp. At December 31, one-half of these goods were included in Seed's ending inventory. Reported selling expenses were $1.1 million and $400,000 for Pard and Seed, respectively. Pard's selling expenses included $50,000 in freight-out costs for goods sold to Seed. What amount of selling expenses should be reported in Pard's consolidated income statement? A. $1,500,000 B. $1,450,000 C. $1,475,000 D. $1,480,000

Answer (D) is correct. Investing activities include the lending of money; the collection of those loans; and the acquisition, sale, or other disposal of (1) loans and other securities that are not cash equivalents and that have not been acquired specifically for resale and (2) property, plant, equipment, and other productive assets. The building had a carrying amount of $52,000 ($100,000 - $48,000), and the loss was $21,000. Hence, the cash inflow from the involuntary conversion (a disposal of property) must have been $31,000 ($52,000 - $21,000).

During the current year, a tornado completely destroyed a building belonging to Holland Corp. The building cost $100,000 and had accumulated depreciation of $48,000 at the time of the loss. Holland received a cash settlement from the insurance company and reported a loss of $21,000. In Holland's current-year cash flow statement, the net change reported in the cash flows from investing activities section should be a A. $21,000 decrease. B. $52,000 decrease. C. $10,000 increase. D. $31,000 increase.

Answer (D) is correct. R&D costs are expensed as incurred. However, legal work in connection with patent applications or litigation and the sale or licensing of patents are specifically excluded from the definition of R&D. Hence, the legal costs of filing a patent should be capitalized. The patent should be amortized over its estimated economic life of 10 years. Amortization for the year equals $1,700 [($34,000 ÷ 10) × (6 ÷ 12)]. Thus, the reported amount of the patent at year end equals $32,300 ($34,000 - $1,700).

During the year just ended, Jase Co. incurred research and development costs of $136,000 in its laboratories relating to a patent that was granted on July 1. Costs of registering the patent equaled $34,000. The patent's legal life is 20 years, and its estimated economic life is 10 years. In its December 31 balance sheet, what amount should Jase report for the patent, net of accumulated amortization? A. $165,000 B. $33,150 C. $161,500 D. $32,300

report it retroactively. make sure you don't apply them to the stocks issued after the split or dividend was declared.

EPS when there is a stock split or stock dividends

premium; premium

Each period, _______________ amortized increases, and interest expense and the carrying amount of the bonds decrease, when amortizing a ____________________.

Answer (A) is correct. Guidance for presentation of earnings per share applies to entities with publicly traded common stock or potential common stock. It also applies to those that have filed or are in the process of filing with a regulatory body in preparation for issuing such securities. Furthermore, if EPS data are presented by nonpublic entities, they must comply with such guidance. EPS disclosures are therefore required only for public companies.

Earnings per share disclosures are required for A. Public entities only. B. Entities with complex capital structures only. C. Public and private entities. D. Entities that change their capital structures during the reporting period.

Answer (D) is correct. The securities were available-for-sale securities after the reclassification, which was at fair value ($530,000 at 6/30/Yr 2). Subsequent unrealized holding gains and losses are excluded from net income and reported in other comprehensive income until realized. Accordingly, the amount reported as net unrealized loss on available-for-sale securities is $40,000 ($530,000 - $490,000 fair value at 12/31/Yr 2).

Fact Pattern: Sun Corp. had investments in trading debt securities costing $650,000. On June 30, Year 2, Sun decided to hold the investments indefinitely and accordingly reclassified them as available-for-sale debt securities on that date. The investments' fair value was $575,000 at December 31, Year 1, $530,000 at June 30, Year 2, and $490,000 at December 31, Year 2. What amount should Sun report as net unrealized loss on available-for-sale securities in its Year 2 other comprehensive income? A. $160,000 B. $45,000 C. $85,000 D. $40,000

Answer (C) is correct. Verifiability is an enhancing qualitative characteristic of relevant and faithfully represented financial information. Information is verifiable (directly or indirectly) if knowledgeable and independent observers can reach a consensus (but not necessarily unanimity) that it is faithfully represented. The existence of an arm's-length transaction between independent interests suggests that the transaction is verifiable.

Financial information is most likely to be verifiable when an accounting transaction occurs that A. Allocates revenues or expense items in a rational and systematic manner. B. Furthers the objectives of the entity. C. Involves an arm's-length transaction between two independent parties. D. Is promptly recorded in a fixed amount of monetary units.

Answer (A) is correct. MD&A is required supplementary information (RSI) that precedes the basic financial statements and provides an overview of financial activities. It is based on currently known facts, decisions, or conditions and includes comparisons of the current and prior years, with an emphasis on the current year, based on government-wide information. Currently known facts are those of which management is aware at the audit report date.

Financial reporting by general-purpose governments includes presentation of management's discussion and analysis as A. A description of currently known facts, decisions, or conditions expected to have significant effects on financial activities. B. Information that may be limited to highlighting the amounts and percentages of change from the prior to the current year. C. Part of the basic financial statements. D. Required supplementary information after the notes to the financial statements.

Answer (C) is correct. Legal fees incurred in a successful defense of a patent should be capitalized and amortized. Legal fees incurred in an unsuccessful defense should be expensed as incurred. Hence, Freya should expense costs for Patent A34 and capitalize costs for Patent B19.

Freya Co. has two patents that have allegedly been infringed by competitors. After investigation, legal counsel informed Freya that it had a weak case for Patent A34 and a strong case in regard to Patent B19. Freya incurred additional legal fees to stop infringement on Patent B19. Both patents have a remaining legal life of 8 years. How should Freya account for these legal costs incurred relating to the two patents? A. Expense costs for both Patent A34 and Patent B19. B. Capitalize costs for both Patent A34 and Patent B19. C. Expense costs for Patent A34 and capitalize costs for Patent B19. D. Capitalize costs for Patent A34 and expense costs for Patent B19.

Answer (D) is correct. The projected benefit obligation (PBO) as of a date is equal to the actuarial present value of all benefits attributed by the pension benefit formula to employee service rendered prior to that date. The PBO is measured using assumptions about future compensation levels.

GAAP relevant to employers' accounting for pensions apply primarily to defined benefit pension plans. It defines the projected benefit obligation as the A. Amount of the adjustment necessary to reflect the difference between actual and estimated actuarial returns. B. Increase in retroactive benefits at the date of the amendment of the plan. C. Present value of benefits accrued to date based on current compensation levels. D. Present value of benefits accrued to date based on future compensation levels.

Answer (C) is correct. When a property dividend is declared, the property is remeasured at its fair value as of the declaration date ($75 - $50 = $25).

Garland Corporation, a public company, has declared a property dividend of one share of its investment in Marlowe, Inc., for every 10 shares of its common stock outstanding. The Marlowe shares were originally purchased by Garland for $50 per share; on the date the dividend was declared, the market value was $75 per share. As a result of this declaration, Garland should recognize A. An appropriate gain or loss based on the market value on the date of distribution. B. A loss of $25 per share to be distributed. C. A gain of $25 per share to be distributed. D. No gain or loss.

Answer (C) is correct. General revenues are revenues not required to be reported as program revenues. They are reported separately after total net (expense) revenue for all functions in the government-wide statement of activities. All taxes, including those levied for a special purpose, are general revenues.

General revenues reported in the government-wide statement of activities A. Exclude taxes levied for a specific purpose. B. Exclude interest and grants. C. Include all taxes. D. Are aggregated with contributions, special and extraordinary items, and transfers in a line item.

Answer (D) is correct. Interperiod equity is an important component of accountability that is fundamental to public administration. Financial resources received during a period should suffice to pay for the services during that period. Moreover, debt should be repaid during the probable period of usefulness of the assets required. Thus, financial reporting should help taxpayers assess whether future taxpayers will have to assume burdens for services already provided. Furthermore, governmental financial reporting also should assist users (e.g., the citizenry and legislative and oversight bodies) in making economic, political, and social decisions. For example, revenue forecasts may help advocates for increased expenditures for education or transportation.

Government financial reporting should provide information to assist users in which situation(s)? Making social and political decisions Assessing whether current-year citizens received services but shifted part of the payment burden to future-year citizens A. II only. B. I only. C. Neither I nor II. D. Both I and II.

-Financial Measurement Resources Focus -Modified Accrual Basis

Governmental fund financial statements are prepared using which of the following: -Current Economic Resources Measurement Focus -Financial Measurement Resources Focus -Modified Accrual Basis -Accrual Basis

Answer (B) is correct. Agency funds are fiduciary funds that may account for certain grants and other financial assistance to be transferred to, or spent on behalf of, secondary recipients (individuals, private organizations, or other governments). The agency fund acts purely as a conduit. It receives the resources and passes them through to the ultimate recipients. However, if the recipient has administrative or direct financial involvement in the program, the pass-through grant is accounted for in an appropriate governmental, proprietary, or trust fund. However, Harland County is merely a conduit for the grant, not a trustee. Thus, it should account for the monies in an agency fund.

Harland County received a $2,000,000 capital grant to be equally distributed among its five municipalities. The grant is to finance the construction of capital assets. Harland had no administrative or direct financial involvement in the construction. In which fund should Harland record the receipt of cash? A. Private purpose trust fund. B. Agency fund. C. Special revenue fund. D. General fund.

The costs of such items acquired or constructed for R&D and *having alternative future uses* (in R&D projects or otherwise) are capitalized as tangible assets and depreciated over their estimated useful lives. The costs of materials, equipment, or facilities acquired or constructed for a particular project and having *no alternative future uses* (and no separate economic values) are R&D costs and are expensed when incurred.

How are R&D costs treated if they have no alternative future uses (and no separate economic values)? What if they do have alternative future uses (in R&D projects or otherwise)

You do not recognize a gain (in all cases but one), but you do recognize a loss. You'll record the new asset at the old assets fair value, debit the loss amount, and then credit the old asset for its book amount

How do you account for a gain or loss during an exchange of assets with NO commercial substance?

Answer (C) is correct. An accounting error results from (1) a mathematical mistake, (2) a mistake in the application of GAAP, or (3) an oversight or misuse of facts existing when the statements were prepared. A change to a generally accepted accounting principle (e.g., accrual-basis accounting) from one that is not (e.g., cash-basis accounting) is an error correction, not an accounting change. An accounting error related to a prior period is reported as a prior-period adjustment by restating the prior-period statements. Restatement requires the same adjustments as retrospective application of a new principle. For example, the carrying amounts of assets, liabilities, and retained earnings at the beginning of the first period reported must be adjusted.

How should a company report its decision to change from a cash-basis of accounting to accrual-basis of accounting? A. As a discontinued operation (net of tax). B. Prospectively, with no amounts restated and no cumulative adjustment. C. As a prior-period adjustment (net of tax), by adjusting the beginning balance of retained earnings. D. As a change in accounting principle, requiring the cumulative effect of the change (net of tax) to be reported in the income statement.

Answer (A) is correct. Imposed nonexchange revenues result from assessments imposed on nongovernmental entities, including individuals, other than assessments on exchange transactions. Examples are property taxes, fines, and forfeitures.

How should a governmental unit classify the fines it assesses? A. Imposed nonexchange revenues. B. Derived tax revenues. C. Government-mandated nonexchange transactions. D. Voluntary nonexchange transactions.

Answer (A) is correct. A bargain purchase is recognized in the consolidated financial statements as an ordinary gain at the acquisition date. A bargain purchase occurs when the net of the acquisition-date fair values of identifiable assets acquired and liabilities assumed exceeds the sum of the acquisition-date fair values of the consideration transferred, any noncontrolling interest recognized, and any previously held equity interest in the acquiree.

How should the acquirer recognize a bargain purchase in a business acquisition? A. As a gain in earnings at the acquisition date. B. As a deferred gain that is amortized into earnings over the estimated future periods benefited. C. As negative goodwill in the statement of financial position. D. As goodwill in the statement of financial position.

Answer (D) is correct. A bargain purchase is recognized in the consolidated financial statements as an ordinary gain at the acquisition date. A bargain purchase occurs when the net of the acquisition-date fair values of identifiable assets acquired and liabilities assumed exceeds the sum of the acquisition-date fair values of the consideration transferred, any noncontrolling interest recognized, and any previously held equity interest in the acquiree.

How should the acquirer recognize a bargain purchase in a business acquisition? A. As negative goodwill in the statement of financial position. B. As goodwill in the statement of financial position. C. As a deferred gain that is amortized into earnings over the estimated future periods benefited. D. As a gain in earnings at the acquisition date.

Answer (D) is correct. The initial capitalized cost of an intangible asset distinct from goodwill is normally the fair value of the consideration given, that is, its purchase price plus incidental costs, such as registration and attorneys' fees. Legal fees incurred in the successful defense of a patent should be capitalized as part of the cost of the patent and then amortized over its remaining useful life. Amortization is appropriate because the useful life of a patent is finite. Legal fees incurred in an unsuccessful defense should be expensed as the costs are incurred. Hence, the cost of Patent A ($40,000 + $5,000) and the legal fees for its successful defense ($20,000) should be capitalized in the amount of $65,000. The costs associated with Patent B should be written off immediately because its unsuccessful defense suggests that no asset exists.

Hy Corp. bought Patent A for $40,000 and Patent B for $60,000. Hy also paid acquisition costs of $5,000 for Patent A and $7,000 for Patent B. Both patents were challenged in legal actions. Hy paid $20,000 in legal fees for a successful defense of Patent A and $30,000 in legal fees for an unsuccessful defense of Patent B. What amounts should Hy capitalize for patents? A. $45,000 B. $162,000 C. $112,000 D. $65,000

traded in an active market.

IFRS difference for intangibles: Intangible assets may be accounted for under either the cost model (as under U.S. GAAP) or the revaluation model. The revaluation model can be applied only if the intangible asset is

Answer (C) is correct. If a capital asset is donated to a governmental unit, it should be recorded at its acquisition value (an entry price). It is the price that would be paid to acquire an asset with equivalent service potential in an orderly market transaction at the acquisition date. In any voluntary nonexchange transaction, assets and revenues are recognized when all eligibility requirements (including time requirements) are met. Because an enterprise fund uses the accrual basis, the resources need not be available.

If a capital asset is donated to a governmental unit, the asset is accounted for in an enterprise fund, and eligibility requirements are met, it should be recorded A. At the lower of the donor's carrying amount or estimated fair value as deferred inflows of resources. B. At the donor's carrying amount as revenue. C. At acquisition value as revenue. D. As a memorandum entry only.

You take the borrowing rate and multiply it by half of the cost of construction that was incurred during the year. 6,000,000 construction cost 300,000 * borrow rate

If a company borrows money to construct a building/asset, how do you classify the interest expense?

Answer (A) is correct. The costs of equipment and facilities that are used for R&D activities and have alternative future uses, whether for other R&D projects or otherwise, are to be capitalized as tangible assets when acquired or constructed. The depreciation of a facility such as a laboratory building used for R&D is an R&D expense.

If a company constructs a laboratory building to be used as a research and development facility, the cost of the laboratory building is matched against earnings as A. Depreciation deducted as part of research and development costs. B. Depreciation or immediate write-off depending on company policy. C. Research and development expense in the period(s) of construction. D. An expense at such time as productive research and development has been obtained from the facility.

Answer (B) is correct. Under the modified approach, infrastructure assets that are part of a network or subsystem of a network (eligible infrastructure assets) need not be depreciated if the government uses an asset management system with certain characteristics and documents that the assets are being preserved approximately at (or above) a condition level established and disclosed by the government. An asset management system should (1) include an updated inventory of eligible infrastructure assets, (2) perform condition assessments and summarize results using a measurement scale, and (3) make annual estimates of the annual amounts needed to maintain the assets at the established condition level.

If a government reports eligible infrastructure assets using the modified approach, A. The assets are not being preserved at or above the established and disclosed condition level. B. No depreciation expense is required to be recognized. C. Expenditures for the assets are capitalized. D. Complete condition assessments must be performed annually.

Answer (C) is correct. The legal costs associated with the successful litigation of a patent defense are capitalized because they benefit future periods, while unsuccessful litigation costs must be expensed as incurred. Thus, only the $100,000 of legal costs associated with unsuccessful outcomes are expensed in Year 3.

In Year 3, a company incurred $500,000 of legal costs defending several patents. Included in that amount was $400,000 of legal costs associated with successful outcomes and $100,000 of legal costs associated with unsuccessful outcomes. What amount of legal costs, if any, should the company expense for Year 3? A. $0 B. $500,000 C. $100,000 D. $400,000

Answer (A) is correct. The legal costs associated with the successful litigation of a patent defense are capitalized because they benefit future periods, while unsuccessful litigation costs must be expensed as incurred. Thus, only the $100,000 of legal costs associated with unsuccessful outcomes are expensed in Year 3.

In Year 3, a company incurred $500,000 of legal costs defending several patents. Included in that amount was $400,000 of legal costs associated with successful outcomes and $100,000 of legal costs associated with unsuccessful outcomes. What amount of legal costs, if any, should the company expense for Year 3? A. $100,000 B. $400,000 C. $500,000 D. $0

Answer (D) is correct. COGS is included in the determination of net income. Cash paid to suppliers, however, is the amount included in determining net cash flows from operating activities. To determine cash paid to suppliers, a two-step adjustment to COGS is necessary. The first step adjusts for the difference between COGS and purchases. The second step adjusts for the difference between purchases and the amounts disbursed to suppliers. The decrease in inventory is therefore subtracted from COGS to arrive at purchases. The decrease in accounts payable is then added to purchases to determine cash paid to suppliers. Accordingly, cash paid to suppliers equals $330,000 ($450,000 COGS - $160,000 + $40,000).

In its Year 6 income statement, Kilm Co. reported cost of goods sold (COGS) of $450,000. Changes occurred in several balance sheet accounts as follows: Inventory $160,000 decrease Accounts payable -- suppliers 40,000 decrease What amount should Kilm report as cash paid to suppliers in its Year 6 cash flow statement, prepared under the direct method?

Answer (D) is correct. If goodwill and another asset group of a reporting unit are tested for impairment simultaneously, the other asset group must be tested for impairment before goodwill. If the asset group was impaired, the impairment loss would be recognized prior to goodwill being tested for impairment.

In one of a company's reporting units, both an asset group and the goodwill are being tested for impairment. Which of the following statements is correct regarding impairment testing and impairment losses? A. If goodwill is impaired, the loss should be recognized prior to testing the other assets for impairment. B. If the other asset group is impaired, the loss should not be recognized prior to goodwill being tested for impairment. C. Impairment testing may be conducted concurrently for the other asset group and goodwill. D. The other asset group should be tested for an impairment loss before goodwill is tested.

Answer (B) is correct. Construction of sidewalks in a new subdivision financed by special assessment bonds is a capital project that results in a general capital asset that will be recognized only in the governmental activities column of the government-wide statement of net position. If the governmental unit is obligated in some manner on the special assessment debt, this capital improvement may be accounted for in the same way as other capital transactions. Thus, the transactions of the construction phase are reported in a capital projects fund. Transactions of the debt service phase are reported in a debt service fund, if one is required. If the government is not obligated, the construction transactions may still be reported in a capital projects fund, but the debt service transactions are recorded in an agency fund.

In what fund type should the proceeds from special assessment bonds issued to finance construction of sidewalks in a new subdivision be reported? A. Special revenue fund. B. Capital projects fund. C. Enterprise fund. D. Agency fund.

In order of liquidity generally, but some indistries may report the most important asset first.

In what order are assets reported on the balance sheet?

COGS / avg inventory

Inventory turnover measures the number of times in a year the total balance of inventory is converted to cash or receivables. Generally, the higher the inventory turnover rate, the more efficient the inventory management of the firm. A high rate may imply that the firm is not carrying excess levels of inventory or inventory that is obsolete.

When the fair value of both assets in a nonmonetary exchange is determinable, the transaction is treated as a monetary exchange. Thus, the asset received is measured at the fair value of the asset given up, and any gain or loss is recognized immediately. This gain or loss is the *difference between the fair value of the asset given up and its carrying amount*

Iona Co. and Siena Co. exchanged goods held for resale with equal fair values. Each will use the other's goods to promote its own products. The retail price of the wickets that Iona gave up is less than the retail price of the wombles received. What gain should Iona recognize on the nonmonetary exchange? A. A gain is not recognized. B. A gain equal to the difference between the retail prices of the wombles received and the wickets. C. A gain equal to the difference between the fair value and the cost of the wickets. D. A gain equal to the difference between the retail price and the cost of the wickets.

A change in the method of depreciation is a change in etimae that is inseperable from a change in accounting principle. It is accounted for prospectively

Is a change in depreciation method a change in principle? How is it accounted for?

DR treasury stock (par X #sh) DR APIC (difference between AP and PV X #sh) DR RE (plug) CR Cash DR treasury stock (pay X #sh) DR APIC (plug) CR Cash

Journal entry for recording the re-acquisition of treasury stock under the Par Value Method 1. acquisition price > original issue price 2. acquisition price < original issue price

DR treasure stock for the cost per stock CR cash

Journal entry for recording the re-acquisition of treasury stock under the cost method

DR treasury stock (cost X # sh) CR cash

Journal entry for recording the re-acquisition of treasury stock under the cost method

Answer (A) is correct. Acquisition-related costs, such as the $320,000 consultant's fee, are expensed as incurred. But exceptions are made for direct issue costs of securities accounted for under other GAAP. Thus, the direct issue costs of equity ($160,000) are debited to additional paid-in capital. The consideration transferred is measured at its fair value of $7,200,000 (100,000 shares of common stock issued × $72). Given that no goodwill or bargain purchase was involved, neither goodwill nor a gain is recognized. Consequently, the identifiable assets acquired and liabilities assumed are recorded at their net amount of $7,200,000. DR. Investment in Sist (100,000 × $72) $7,200,000 CR. C/S (100,000 × $40) $4,000,000 CR. APIC (plug) 3,200,000 DR. Business combination expense $ 320,000 DR. Additional paid-in capital 160,000 CR. Cash $ 480,000

Just Issue the Stock On August 31, Planar Corp. exchanged 100,000 shares of its $40 par value common stock for all of the net assets of Sistrock Co. The fair value of Planar's common stock on August 31 was $72 per share. Planar paid a fee of $320,000 to the consultant who arranged this acquisition. Direct costs of registering and issuing the equity securities amounted to $160,000. No goodwill or bargain purchase was involved in the acquisition. At what amount should Planar record the acquisition of Sistrock's net assets? A. $7,200,000 B. $7,360,000 C. $7,680,000 D. $7,520,000

Answer (A) is correct. The exchange of debt for a long-lived asset does not involve a cash flow. It is therefore classified as a noncash financing and investing activity.

Kelli Company acquired land by assuming a mortgage for the full acquisition cost. This transaction should be disclosed on Kelli's statement of cash flows as a(n) A. Noncash financing and investing activity. B. Operating activity. C. Investing activity. D. Financing activity.

Answer (C) is correct. With certain exceptions, e.g., (1) money market investments, (2) external investment pools, (3) life insurance contracts, and (4) equity-method investments in common stock, investments should be measured at fair value. Thus, land and other real estate held as investments by (1) endowments, including permanent endowments and term endowments, and (2) permanent funds should be reported at fair value at the reporting date. Changes in fair value should be reported as investment income. Fair value is an exit price. It is the price that would be received to sell an asset, or paid to transfer a liability, in an orderly transaction between market participants at the measurement date.

Land and other real estate held as investments by endowments in a government's permanent fund should be reported at A. The lower of cost and net realizable value. B. Fair value less costs of disposal. C. Fair value. D. Historical cost.

pay an additional amount in order to receive the stock.

Like convertible debt, stock warrants allow a bondholder to obtain shares of common stock. Unlike convertible debt, warrants require the bondholder to

DR Cash 511,925 DR Unamortized Debt Issue Cost 40000 CR Bonds Payable 500,000 CR Premium on Bonds 51,925

Lime Industries, Inc., issued 500 9%, 4-year, $1,000 bonds when the prevailing interest rate was 6%. The present value of the face amount is $396,000, and the present value of the cash interest is $155,925. Bond issue costs to bring the debt to market were $40,000. What is the journal entry to record the bonds issued? what's the journal entry for issuing the bonds?

Answer (A) is correct. Capital outlays financed from general obligation bond proceeds should be accounted for in a capital projects fund. Consequently, the $5,000,000 in bond proceeds to be used to finance the major capital project should be credited to other financing sources in a capital projects fund. The $50,000 issuance premium to be used for payment of principal and interest should be recorded directly as an other financing source in the debt service fund. A debt service fund must be used if legally required or if resources are being accumulated for payments due in future periods. Moreover, the debt service fund is included in each answer choice. The $5,000,000 of general obligation bonds should be reported only in the governmental activities column of the government-wide statement of net position. This treatment of an unmatured long-term liability is appropriate if it is not directly related to or expected to be paid from proprietary or fiduciary funds.

Lisa County issued $5,000,000 of general obligation bonds at 101 to finance a major capital project. The $50,000 premium was to be used for payment of principal and interest. This transaction should be accounted for in the A. Capital projects fund, debt service fund, and government-wide financial statements. B. Debt service fund only. C. Debt service fund and government-wide financial statements only. D. Capital projects fund and debt service fund only.

Answer (A) is correct. The compensated absences liability should be calculated based on the pay or salary rates in effect at the balance sheet date. However, the employer might pay employees for their compensated absences at other than their pay or salary rates. For example, payment might be at a lower amount established by contract, regulation, or policy. That other rate determined at the balance sheet date should be used to calculate the liability.

Lys City reports a general long-term compensated absences liability in its financial statements. The salary rate used to calculate the liability should normally be the rate in effect A. At the balance sheet date. B. When the compensated absences are to be paid. C. When the compensated absences were earned or are to be paid, or at the balance sheet date, whichever results in the lowest amount. D. When the unpaid compensated absences were earned.

Answer (C) is correct. The first step in preparing government-wide financial statements is to convert the modified accrual data in the governmental fund financial statements. The conversion is performed at the total governmental fund summary level, not at the individual fund level. Bond proceeds increase current financial resources in the governmental funds. However, issuing debt increases long-term liabilities in the statement of net position with no effect on the statement of activities. Also, grant receipts are reported in the same way in governmental fund financial statements and government-wide financial statements. Thus, the change in net position of governmental activities in the government-wide statement of activities is $50,000,000 less than the net change in governmental fund balances.

Meen County issued a $50,000,000 bond at its face amount and received a $1,000,000 grant for improving parks. What is the net effect on the reconciliation of (1) the net change in governmental fund balances to (2) the change in net position of governmental activities in the government-wide statement of activities? A. $0 B. $1,000,000 increase. C. $50,000,000 decrease. D. $51,000,000 decrease.

Answer (D) is correct. The economic resources measurement focus and the accrual basis of accounting are required in the proprietary funds financial statements. Enterprise funds and internal service funds are proprietary funds. Capital assets of internal service funds are reported in the proprietary funds statement of net position on the same basis as business entities. The printing equipment therefore is reported at the cost of acquisition and installation, or $205,000 ($200,000 price + 5,000 cost of installation).

Meen County uses an internal service fund to account for printing services provided to other departments. Meen County paid $200,000 for new printing equipment, a $5,000 fee for installation of new printing equipment, and $7,000 for ink. What amount should the internal service fund report as the cost of the new printing equipment on the proprietary funds statement of net position? A. $0 B. $212,000 C. $200,000 D. $205,000

providing information for the entity as a whole

NFP measurement focus

1. education in general- faculty salaries, student services, research 2. auxiliary enterprise- cafeteria, dorms, bookstores, athletic 3. support- management, administrative, fundraising

NFP universities have three types of expenses

Answer (A) is correct. Individual items or collections of works of art, historical treasures, and similar assets ordinarily must be capitalized. However, if a collection is (1) held in furtherance of public service and not for gain; (2) protected, preserved, cared for, and kept unencumbered; and (3) subject to a policy that sale proceeds are to be used to obtain other collection items, capitalization is not required. If capitalized collections or individual items are exhaustible, for example, because their useful lives are reduced by display, educational, or research uses, they must be depreciated.

Nack City received a donation of a valuable painting. Nack planned to add the painting to its collection and display it in the protected exhibition area of city hall. Nack had a policy that if such donated art works were sold, the proceeds would be used to acquire other items for its collections. Which of the following would be correct regarding the donated painting? A. May be capitalized, but it is not required, and depreciation is not required. B. May be capitalized, but it is not required, and it must be depreciated. C. Must be capitalized but not depreciated. D. Must be capitalized and depreciated.

NRV = Sales price - selling cost

Net realizable value when you are valuing inventory =

Answer (B) is correct. Organization costs are those incurred in the formation of a business entity. For financial accounting purposes, nongovernmental entities must expense all start-up and organization costs as incurred. Thus, the legal fees should be expensed because they are organization costs. Fees for an initial stock offering are customarily treated as a reduction in the proceeds rather than as organization costs, and exploration costs and purchases of mineral rights are capitalizable items that are not organization costs.

Neue Co. incurred the following costs during its first year of operations: -Legal fees for incorporation and other related matters $55,000 -Underwriters' fees for initial stock offering 40,000 -Exploration costs and purchases of mineral rights 60,000 Neue had no revenue during its first year of operation. What amount must Neue expense as organizational costs? A. $95,000 B. $55,000 C. $100,000 D. $155,000

Answer (A) is correct. The trademark is expected to be renewed indefinitely. Thus, it has an indefinite useful life and is not amortized. Amortization is recorded only for an intangible asset with a finite useful life.

Northstar Co. acquired a registered trademark for $600,000. The trademark has a remaining legal life of 5 years but can be renewed every 10 years for a nominal fee. Northstar expects to renew the trademark indefinitely. What amount of amortization expense should Northstar record for the trademark in the current year? A. $0 B. $120,000 C. $40,000 D. $15,000

reduce the carrying amount of capitalized software costs. Once software costs reach $0, any further net proceeds are recognized as revenue.

Occasionally, software developed for internal use will be sold to outside parties. The net proceeds from these sales

Answer (B) is correct. Acquisition-related costs ($160,000) are normally expensed as incurred. But costs of registering and issuing equity securities ($80,000) are a charge to additional paid-in capital. The fair value of the consideration transferred in this 100% acquisition equals the fair value of the net assets acquired, given no goodwill or bargain purchase. This amount (a net debit) is $3,600,000 (100,000 shares × $36).

On August 31 of the current year, Pine Corp. issued 100,000 shares of its $20 par value common stock for all of the net assets of Sap, Inc., in a business combination. The fair value of Pine's common stock on the acquisition date was $36 per share. Pine paid a fee of $160,000 to the consultant who arranged this acquisition. Costs of registering and issuing the equity securities amounted to $80,000. No goodwill or gain on a bargain purchase was involved. What amount should Pine record for the net assets acquired? A. $3,840,000 B. $3,600,000 C. $3,760,000 D. $3,680,000

Answer (D) is correct. Acquisition-related costs, such as the $320,000 consultant's fee, are expensed as incurred. But exceptions are made for direct issue costs of securities accounted for under other GAAP. Thus, the direct issue costs of equity ($160,000) are debited to additional paid-in capital. The consideration transferred is measured at its fair value of $7,200,000 (100,000 shares of common stock issued × $72). Given that no goodwill or bargain purchase was involved, neither goodwill nor a gain is recognized. Consequently, the identifiable assets acquired and liabilities assumed are recorded at their net amount of $7,200,000. The journal entries are

On August 31, Planar Corp. exchanged 100,000 shares of its $40 par value common stock for all of the net assets of Sistrock Co. The fair value of Planar's common stock on August 31 was $72 per share. Planar paid a fee of $320,000 to the consultant who arranged this acquisition. Direct costs of registering and issuing the equity securities amounted to $160,000. No goodwill or bargain purchase was involved in the acquisition. At what amount should Planar record the acquisition of Sistrock's net assets? A. $7,360,000 B. $7,520,000 C. $7,680,000 D. $7,200,000

Answer (A) is correct. Organization costs are those incurred in the formation of a business entity. Under the federal tax code, organization and start-up costs must be capitalized and amortized over a period of not less than 15 years. However, for financial accounting purposes, nongovernmental entities must expense all start-up and organization costs as incurred.

On January 1, Trilite Corp. incurred organization costs of $24,000. Trilite is amortizing these costs so as to obtain the maximum allowable deduction for federal income tax purposes. For financial accounting purposes, what portion of the organization costs will Trilite defer to subsequent years? A. $0 B. $19,200 C. $4,800 D. $23,400

Answer (C) is correct. The costs of materials, equipment, or facilities that are acquired or constructed for a particular R&D project and that have no alternative future uses and therefore no separate economic values are R&D costs when incurred. R&D costs are expensed in full when incurred.

On January 1, Year 1, Jambon purchased equipment for use in developing a new product. Jambon uses the straight-line depreciation method. The equipment could provide benefits over a 10-year period. However, the new product development is expected to take 5 years, and the equipment can be used only for this project. Jambon's Year 1 expense equals A. One-fifth of the cost of the equipment. B. One-tenth of the cost of the equipment. C. The total cost of the equipment. D. Zero.

Answer (A) is correct. Revenue for sales-based royalties from licensed intellectual property (e.g., a patent) is recognized when the subsequent sale occurs. Boulder is guaranteed at least $100,000 over the life of the patent. However, it must recognize the royalty revenue when sales occur over the 4-year useful life of the patent. For the year, Boulder should report $30,000 ($300,000 × 10%) of the patent revenue.

On January 2, Boulder Co. assigned its patent to Castle Co. for royalties of 10% of patent-related sales. The assignment is for the remaining 4 years of the patent's life. Castle guaranteed Boulder a minimum royalty of $100,000 over the life of the patent and paid Boulder $50,000 against future royalties during the year. Patent-related sales for the year were $300,000. In its income statement for the year, what amount should Boulder report as royalty revenue? A. $30,000 B. $100,000 C. $25,000 D. $50,000

Answer (C) is correct. The cash equivalent price of the machinery (present value), reduced by the salvage value, equals the depreciable base. The excess of the total cash to be paid over the cash equivalent price of the machinery will be recognized as interest expense, not depreciation. Accordingly, straight-line depreciation is $10,500 [($110,000 cash equivalent price - $5,000 salvage value) ÷ 10 years].

On January 2, Lem Corp. bought machinery under a contract that required a down payment of $10,000, plus 24 monthly payments of $5,000 each, for total cash payments of $130,000. The cash equivalent price of the machinery was $110,000. The machinery has an estimated useful life of 10 years and estimated salvage value of $5,000. Lem uses straight-line depreciation. In its income statement for the year, what amount should Lem report as depreciation for this machinery? A. $11,000 B. $12,500 C. $10,500 D. $13,000

Answer (B) is correct. An impairment loss should be recognized because the carrying amount is not recoverable. This loss is measured by the excess of the carrying amount over the fair value. Accordingly, the impairment loss recognized in Year 4 equals the carrying amount because the fair value is $0. The amortization for each year was based on the straight-line method because the pattern of consumption of the economic benefits of the intangible asset is not reliably determinable. Thus, the annual amortization amount given a 10-year useful life was $9,000 ($90,000 ÷ 10), and the accumulated amortization expense at the beginning of Year 4 was $27,000 ($9,000 × 3). Consequently, the loss from the write-off is the carrying amount of $63,000 ($90,000 - $27,000).

On January 2, Year 1, Lava, Inc., purchased a patent for a new consumer product for $90,000. At the time of purchase, the patent was valid for 15 years; however, the patent's useful life was estimated to be only 10 years due to the competitive nature of the product. On December 31, Year 4, the product was permanently withdrawn from sale under governmental order because of a potential health hazard in the product. What amount should Lava charge against income during Year 4, assuming amortization is recorded at the end of each year but the pattern of consumption of the economic benefits of the patent is not reliably determinable? A. $72,000 B. $63,000 C. $9,000 D. $54,000

Answer (B) is correct. At 1/3/Year 4, the carrying amount of the computer equipment should be written down to $70,000. This $70,000 is expected to be recovered over the 3.5-year remaining useful life of the equipment. Under the straight-line method, the depreciation expense for the year ending 12/31/Year 4 is $20,000 [($70,000 ÷ 42 months) × 12 months]. Thus, the carrying amount in the year-end balance sheet should be $50,000 ($70,000 - $20,000).

On July 1, Year 1, Rey Corp. purchased computer equipment at a cost of $360,000. This equipment was estimated to have a 6-year life with no residual value and was depreciated by the straight-line method. On January 3, Year 4, Rey determined that this equipment could no longer process data efficiently, that its value had been permanently impaired, and that $70,000 could be recovered over the remaining useful life of the equipment. What carrying amount should Rey report on its December 31, Year 4, balance sheet for this equipment? A. $0 B. $50,000 C. $70,000 D. $150,000

Answer (D) is correct. When an intangible asset is acquired in an exchange transaction, initial recognition is at the fair value of the more clearly evident of the consideration given or the asset acquired. The fair value of the assets given in return for the patent was $80,000 (2,000 shares of stock × $40 per share fair value).

On June 30, Year 5, Finn, Inc., exchanged 2,000 shares of Edlow Corp. $30 par-value common stock for a patent owned by Bisk Co. The Edlow stock was acquired in Year 1 at a cost of $50,000. At the exchange date, Edlow common stock had a fair value of $40 per share, and the patent had a net carrying amount of $100,000 on Bisk's books. Finn should record the patent at A. $60,000 B. $50,000 C. $100,000 D. $80,000

Answer (B) is correct. Current assets exclude cash and claims to cash that are segregated for the liquidation of long-term debts. None of the assets in the sinking fund offset maturing debt that has been set up as a current liability. Consequently, the entire balance is a noncurrent asset.

On March 1, Year 1, a company established a sinking fund in connection with an issue of bonds due in Year 13. At December 31, Year 5, the independent trustee held cash in the sinking-fund account representing the annual deposits to the fund and the interest earned on those deposits. How should the sinking fund be reported in the company's classified balance sheet at December 31, Year 5? A. Only the accumulated deposits should appear as a noncurrent asset. B. The entire balance in the sinking-fund account should appear as a noncurrent asset. C. The cash in the sinking fund should appear as a current asset. D. The entire balance in the sinking-fund account should appear as a current asset.

Answer (C) is correct. When the outcome of the contract is not reasonably measurable, but the costs incurred in satisfying the performance obligation are expected to be recovered, revenue must be recognized only to the extent of the costs incurred. Revenue recognized is based on a zero profit margin until the entity can reasonably measure the outcome of the performance obligation. Because the costs incurred are expected to be recovered, revenue is recognized equal to the costs incurred of $450,000 in Year 1.

On October 1, Year 1, Little Co. enters into a contract with a customer to build a factory on the customer's land for $1,000,000. The construction of the factory is expected to be completed at the end of Year 5. Based on Little's accounting policies, the progress toward completion of the factory is measured using the input method based on costs incurred. During Year 1, Little incurred $450,000 in costs in respect to this contract and billed the customer for $600,000. At the end of Year 1, Little cannot reasonably estimate the total expected costs of the construction and cannot reasonably estimate the progress toward completion of the factory. However, Little expects to recover the costs incurred in the construction. What amount of revenue from this contract will be recognized in Little's Year 1 income statement? A. $150,000 B. $600,000 C. $450,000 D. $0

Answer (B) is correct. When a change in the tax law or rates occurs, the effect of the change on a deferred tax liability or asset is recognized as an adjustment in the period that includes the enactment date of the change. The adjustment is allocated to income from continuing operations in the first financial statements issued for the period that includes the enactment date.

On September 15, Year 4, the county in which Spirit Company operates enacted changes in the county's tax law. These changes are to become effective on January 1, Year 5. They will have a material effect on the deferred tax amounts that Spirit reported. In which of the following interim and annual financial statements issued by Spirit should the effect of the changes in tax law initially be reported? A. The annual financial statements for the year ending December 31, Year 5. B. The interim financial statements for the 3-month period ending September 30, Year 4. C. The annual financial statements for the year ending December 31, Year 4. D. The interim financial statements for the 3-month period ending March 31, Year 5.

The payment of cash to purchase equipment used in production is a cash outflow from an investing activity.

Operating, investing, or financing? The payment of cash for the purchase of additional equipment needed for current production.

Answer (D) is correct. On the business combination date, identifiable assets acquired, liabilities assumed, and any noncontrolling interest in the acquiree must be measured at acquisition-date fair value. Even though Gravel Co. expensed the R&D costs in its separate financial statements, on the business combination date, the in-process R&D is an identifiable asset. Thus, it must be measured and reported in the consolidated financial statements at its acquisition-date fair value.

Park, Inc., acquired 100% of Gravel Co.'s net assets. On the acquisition date, Gravel's accounting records reflected $50,000 of costs associated with in-process research and development activities. The fair value of the in-process research and development activities was $400,000. Park's consolidated intangible assets will increase by what amount, if any, as a result of the acquisition of the in-process research and development activities? A. $50,000 B. $350,000 C. $0 D. $400,000

Answer (A) is correct. Acquisition-related costs, such as finder's fees, professional (e.g., legal) and consulting fees, and general administrative costs (e.g., for an acquisitions department), are expensed as incurred. The one exception is for the issuance costs of debt or equity securities. These are accounted for under other GAAP. Thus, direct issuance costs of equity (underwriting, legal, accounting, tax, registration, etc.) are debited to additional paid-in capital. Accordingly, the amount recorded in APIC for the combination should be $1,365,000 {[200,000 shares × ($12 market value - $5 par value)] - $35,000 direct issuance costs}.

Pendragon Co. issues 200,000 shares of $5 par value common stock to acquire Squire Co. in a business combination. The market value of Pendragon's common stock is $12. Legal and consulting fees incurred in relationship to the combination are $110,000. Direct registration and issuance costs for the common stock are $35,000. What should be recorded in Pendragon's additional paid-in capital (APIC) for this business combination? A. $1,365,000 B. $1,255,000 C. $1,400,000 D. $1,545,000

Answer (D) is correct. Acquisition-related costs, such as finder's fees, professional (e.g., legal) and consulting fees, and general administrative costs (e.g., for an acquisitions department), are expensed as incurred. The one exception is for the issuance costs of debt or equity securities. These are accounted for under other GAAP. Thus, direct issuance costs of equity (underwriting, legal, accounting, tax, registration, etc.) are debited to additional paid-in capital. Accordingly, the amount recorded in APIC for the combination should be $1,365,000 {[200,000 shares × ($12 market value - $5 par value)] - $35,000 direct issuance costs}.

Pendragon Co. issues 200,000 shares of $5 par value common stock to acquire Squire Co. in a business combination. The market value of Pendragon's common stock is $12. Legal and consulting fees incurred in relationship to the combination are $110,000. Direct registration and issuance costs for the common stock are $35,000. What should be recorded in Pendragon's additional paid-in capital (APIC) for this business combination? A. $1,400,000 B. $1,545,000 C. $1,255,000 D. $1,365,000

Answer (C) is correct. Acquisition-related costs, such as finder's fees, professional (e.g., legal) and consulting fees, and general administrative costs (e.g., for an acquisitions department), are expensed as incurred. The one exception is for the issuance costs of debt or equity securities. These are accounted for under other GAAP. Thus, direct issuance costs of equity (underwriting, legal, accounting, tax, registration, etc.) are debited to additional paid-in capital. Accordingly, the amount recorded in APIC for the combination should be $1,365,000 {[200,000 shares × ($12 market value - $5 par value)] - $35,000 direct issuance costs}.

Pendragon Co. issues 200,000 shares of $5 par value common stock to acquire Squire Co. in a business combination. The market value of Pendragon's common stock is $12. Legal and consulting fees incurred in relationship to the combination are $110,000. Direct registration and issuance costs for the common stock are $35,000. What should be recorded in Pendragon's additional paid-in capital (APIC) for this business combination? A. $1,545,000 B. $1,255,000 C. $1,365,000 D. $1,400,000

Defined benefit plan- benefits that will be paid out at retirement. SIR PAT 1. S- Service Cost + 2. I- Interest on projected benefit obligation + 3. R- Return on plan assets - 4. P- Prior service costs +/- 5. A- Actuarial G/L +/- 6. T- Transition Asset/Liability +/-

Pension Plans- Defined benefit plan

-Current Economic Resources Measurement Focus -Accrual Basis The economic resources measurement focus and the accrual basis of accounting are required in the proprietary fund financial statements. The economic resources measurement focus differs from the shorter-term flow-of-current-financial-resources approach used in governmental funds. It measures revenues and expenses in the same way as in commercial accounting. The emphasis is on a longer-range measure of revenues earned or levied (and accrued immediately if measurable). Moreover, the economic resources model focuses on cost of services. The accrual basis of accounting recognizes most transactions when they occur, regardless of when cash is received or paid.

Proprietary fund financial statements are prepared using which of the following: -Current Economic Resources Measurement Focus -Financial Measurement Resources Focus -Modified Accrual Basis -Accrual Basis

excludes inventory and pre-paids CA/CA

Quick ratio

no

R&D cost? Equipment acquired for use in various research and development projects

no

R&D? Activity, including design and construction engineering, related to the construction, relocation, rearrangement, or start-up of facilities or equipment other than pilot plants and facilities or equipment whose sole use is for a particular R&D project

no

R&D? Adaptation of an existing capability to a particular requirement or customer's need as part of a continuing commercial activity

yes

R&D? Conceptual formulation and design of possible product or process alternatives

yes

R&D? Design, construction, and testing of preproduction prototypes and models

yes

R&D? Engineering activity required to advance the design of a product until it meets specific functional and economic requirements and is ready for manufacture

no

R&D? Engineering follow-through in an early phase of commercial production

no

R&D? Legal work in connection with patent applications or litigation and the sale or licensing of patents

yes

R&D? Modification of the formulation or design of a product or process

no

R&D? Quality control during commercial production, including routine testing of products

no

R&D? Routine, ongoing efforts to refine, enrich, or otherwise improve upon the qualities of an existing product

yes

R&D? Searching for applications of new research findings or other knowledge

yes

R&D? Testing in search for, or evaluation of, product or process alternatives

Answer (C) is correct. An expense is not accrued prior to the occurrence of the event for which an entity self-insures. The fair value of the property diminishes only if the event actually occurs. But an appropriation of retained earnings is acceptable to disclose the self-insurance policy if, when a fire loss occurs, the entry appropriating retained earnings is reversed, and the loss is debited to income of the period of loss and not to retained earnings.

Sanders Company effects self-insurance against loss from fire by appropriating an amount of retained earnings each year equal to the amount that would otherwise be paid out as fire insurance premiums. According to current accounting literature, the procedure used by Sanders is A. Acceptable provided that fire losses are charged against the appropriation. B. Acceptable if the amount is shown outside the equity section of the balance sheet. C. Acceptable provided that fire losses are not charged against the appropriation. D. Prohibited for external reporting purposes.

Costs incurred during the preliminary project stage (planning, evaluation) are expensed as incurred. Costs incurred during the application development stage (coding, testing) are capitalized as computer software costs. They include (a) external direct costs of materials and services, (b) payroll costs directly associated with the project, and (c) interest costs associated with the project.

Software to Be Used Internally: What costs are expensed? What costs are capitalized?

capitalized as computer software costs.

Software to be marketed: Costs incurred after technological feasibility is established (coding, testing, producing product masters) are

expensed as incurred. Thus, they are treated as R&D costs. Technological feasibility is established when either a detailed program design is complete or the entity has created a working model.

Software to be marketed: Costs incurred before technological feasibility is established (coding, testing, etc.) are

3. Other coding costs after establishment of technological feasibility 4. Other testing costs after establishment of technological feasibility 5. Costs of producing product masters for training materials costs 1 and 2 are expensed and costs 6 and 7 are capitalized as inventory

Software to be marketed: which costs are capitalized? 1. Completion of detail program design 2. Costs incurred for coding and testing to establish technological feasibility 3. Other coding costs after establishment of technological feasibility 4. Other testing costs after establishment of technological feasibility 5. Costs of producing product masters for training materials 6. Duplication of computer software and training materials from product masters (1,000 units) 7. Packaging product (500 units)

a liability; a contract to perform R&D for others

Sometimes an entity's R&D is funded wholly or partly by others. 1. If the entity is obligated to repay any of the funds provided by the other party regardless of the outcome of the project, it recognizes ___________________. 2. If repayment depends solely on the results of the R&D having future economic benefit, the entity accounts for its obligation as _________________.

false: Not all qualifying investments must be classified as cash equivalents. An entity should consistently apply a policy for classifying cash equivalents. For example, an entity with operations that primarily involve investing in short-term, highly liquid investments may choose not to treat them as cash equivalents.

T/F: All qualifying investments must be classified as cash equivalents.

False: Any change in the policy for classifying cash equivalents is a change in accounting principle that requires retrospective application.

T/F: Any change in the policy for classifying cash equivalents is applied prospectively.

FALSE Minimum rental payments are the periodic amounts owed by the lessee, minus any executory costs (such as insurance, maintenance, or taxes) that will be paid by the lessor.

T/F: Executory costs of a lease include such items as insurance, maintenance, and taxes to be paid by the lessor. They are added to the minimum rental payments.

TRUE: Financing activities include (1) issuance of stock, (2) payment of dividends, (3) treasury stock transactions, (4) incurrence of debt, (5) repayment or other settlement of debt obligations, and (6) the exercise of share options resulting in excess tax benefits.

T/F: Financing activities include issuance of stock and payment of dividends.

False: If an entity invests its cash in cash equivalents, it must use a descriptive term such as "cash and cash equivalents." Otherwise, the term "cash" is acceptable. Terms such as "funds" or "quick assets" may not be used.

T/F: If an entity that invests in cash equivalents may use a descriptive term such as "funds" in its statement of cash flows.

FALSE If the lessor's implicit rate is unknown to the lessee, the lessor and the lessee may use different rates. Thus, if the lessee and lessor use different rates, one might recognize an operating lease and the other might recognize a capital lease.

T/F: If the lessor recognizes the lease as a capital lease, the lessee must also recognize the lease as a capital lease.

False: *Financing activities* include cash payments by the lessee for a reduction of the outstanding liability related to a capital lease. The cash payment for the interest portion of a capital lease is classified as an operating activity.

T/F: Operating activities include cash payment for the liability portion of a capital lease.

TRUE: Cash outflows from operating activities include cash payments to buy materials for manufacture of goods for resale, including principal payments on accounts and notes payable to suppliers. They also include cash payments to (1) other suppliers and employees for other goods and services; (2) governments for taxes, fees, and penalties; and (3) creditors for interest.

T/F: Operating cash outflows include principal payments on notes to suppliers.

FALSE: Under *IFRS*, an entity that is not a financial institution may classify interest and dividends received as cash inflows from *operating and investing activities*. Under GAAP, interest and dividends paid may be classified as cash inflows from *operating and financing activities*

T/F: Under IFRS, an entity that is not a financial institution may classify interest and dividends received as cash inflows from operating and financing activities.

According to IFRS, if an entity is not a financing institution, interest paid and interest received may be classified as financing or investing cash flows, respectively, because they are costs of obtaining financial resources or returns on an investment, respectively.

T/F: Under IFRS, interest received or paid must be classified as a cash flow from operating activities.

TRUE: Cash inflows from operating activities include receipts from collection or sale of accounts and notes resulting from sales to customers. They also include cash receipts in the form of interest and dividends, that is, returns on loans, other debt instruments of other entities, and equity securities.

T/F: Under U.S. GAAP, operating cash inflows include cash interest and dividends received.

TRUE

T/F: Under the direct method, *depreciation expense* for the period must be *added* to net income to determine the net cash flow from operating activities.

TRUE: Under the indirect method (also called the reconciliation method), the net income for the period must be adjusted to determine the net cash flow from operating activities. One of those adjustments is the effect of noncash revenue and expenses that were included in net income, such as depreciation and amortization expenses, impairment losses, etc. Depreciation expense is a noncash expense included in net income. Thus, it must be added to net income to determine the net cash flow from operating activities. Your answer is correct. Under the indirect method (also called the reconciliation method), the net income for the period must be adjusted to determine the net cash flow from operating activities. One of the adjustments is the effect of items included in net income whose cash effects relate to investing or financing cash flows. Gain on sale of property is an item included in net income. Its cash effect is related to investing activities. Thus, it must be subtracted from net income to determine the net cash flow from operating activities.

T/F: Under the indirect method, *depreciation expense and a decrease in inventory* for the period must be *added* to net income to determine the net cash flow from operating activities. Under the indirect method, a *gain on disposal of a property* is *subtracted* from the net income to determine the cash flow from operating activities.

False. A commitment to acquire goods in the future is not recorded at the time of the agreement. GAAP require accrual of a loss in the current year's income statement on goods subject to a firm purchase commitment if the market price of the goods declines below the commitment price.

T/F? A commitment to acquire goods in the future is recorded at the time of the agreement.

False. Shorter, not longer

T/F? A receivable is a current asset if it is reasonably expected to be collected within the longer of 1 year or the entity's normal operating cycle. Otherwise, it should be classified as noncurrent.

TRUE: Under the indirect method, an entity must provide a supplemental disclosure of the amounts paid for (1) interest expense (net of amount capitalized) and (2) income taxes.

T/F? Under the indirect method of presenting operating cash flows, the amount of cash paid for interest and income taxes must be disclosed in supplementary information.

Answer (C) is correct. A permanent fund is a governmental fund that accounts for resources that are restricted to the use of earnings for the benefit of the government and its citizens.

Tang City received land from a donor who stipulated that the land must remain intact, but any income generated from the property may be used for general government services. In which fund should Tang City record the donated land? A. Private-purpose trust. B. Agency. C. Permanent. D. Special revenue.

direct; direct

The ______________ method converts the accrual-basis amounts in the income statement to the cash basis. It then presents certain minimum cash amounts in the operating section. However, if the ___________ method is used, a reconciliation of net income and net operating cash flow must be reported in a separate schedule.

allowance for doubtful accounts method

The _________________ attempts to match bad debt expense with the related revenu

direct write-off method

The ___________________ expenses bad debts when they are determined to be uncollectible. It is not acceptable under GAAP

Answer (C) is correct. When a budget is initially recorded, appropriations control is credited for the total authorized and estimated expenditures for the fiscal year. It is debited at year end when the budgetary accounts are closed.

The appropriations control account of a governmental unit is debited when A. The budget is recorded. B. Supplies are purchased. C. The budgetary accounts are closed. D. Expenditures are recorded.

Answer (C) is correct. Zero-coupon bonds do not pay periodic interest. The bonds are sold at a discount from their face value, and the investors do not receive interest until the bonds mature. The issuer does not have to make annual cash outlays for interest. However, the discount must be amortized annually and reported as interest expense.

The best advantage of a zero-coupon bond to the issuer is that the A. Interest can be amortized annually by the APR method and need not be shown as an interest expense to the issuer. B. Bond requires no interest income calculation to the holder or issuer until maturity. C. Interest can be amortized annually on a straight-line basis but is a noncash outlay. D. Bond requires a low issuance cost.

Answer (A) is correct. The standalone selling price is the price at which an entity would sell a promised good or service separately to a customer. The best evidence of a standalone selling price is the observable price of a good or service when it is sold separately in similar circumstances and to similar customers (e.g., a contractually stated price or list price of a good or service).

The best evidence of a standalone selling price of a promised good or service to a customer is A. An observable price. B. Competitor's selling price. C. Expected cost. D. Expected cost plus an appropriate margin.

they will benefit future periods; expensed as incurred.

The costs of successful litigation are capitalized because _________________________. They are amortized over the shorter of the remaining legal life or the estimated useful life of the patent. The costs of unsuccessful litigation (damages, attorneys' fees) are _______________________.

Answer (A) is correct. General capital assets are all capital assets not reported in the proprietary funds or the fiduciary funds. Purchased capital assets are reported at historical cost, including ancillary charges (e.g., for freight and site preparation) only in the governmental activities column of the government-wide statement of net position.

The government-wide financial statements report purchased capital assets A. At historical cost, including ancillary charges. B. Only in the notes if they are donated. C. At acquisition value. D. In the general fixed assets account group.

Answer (B) is correct. The statement of activities presents operations in a format that displays net (expense) revenue for each function. The net (expense) revenue for each governmental or business-type function equals expenses minus program revenues. The minimum levels of detail for activities of governmental funds and enterprise funds are by function and by different identifiable activities, respectively.

The government-wide statement of activities reports A. Activities accounted for in governmental funds at least at the segment level of detail. B. Net (expense) revenue for each function equal to expenses minus program revenues. C. Activities accounted for in the enterprise funds at least at the fund level of detail. D. Net (expense) revenue for each function equal to expenses minus general revenues.

Answer (C) is correct. Fair value is a better measure of a government's investments than cost because it helps to assess investment management and performance and financial position. Furthermore, changes in fair value should be recognized as investment income in the year they occur because they are as relevant as other investment earnings, e.g., dividends and interest, to the foregoing assessment. This approach also avoids the possibility that investment sales might be timed to distort results.

The majority of state and local governmental entities, including external investment pools, report most investments at fair value. Assuming that a governmental entity reports investments at fair value, changes in the fair value of the investments should be recognized as A. A direct change in a component of equity. B. A component of the operating statement below the excess of revenues over expenditures, or net income. C. Investment income in the operating statement when they occur. D. Investment income in the operating statement when they are realized.

Answer (C) is correct. The basis of accounting is the timing of the recognition in the financial records of economic events or transactions. The basis of accounting of a fund depends on its measurement focus. This measurement focus is what is being measured or tracked by information in the financial statements. The modified accrual basis of accounting is used to report the governmental fund financial statements. The measurement focus is on current financial resources, that is, on determining the financial position and changes in it. The reporting elements are sources, uses, and balances of current financial resources.

The measurement focus of governmental fund accounting is on which of the following? A. Economic resources. B. Cash. C. Current financial resources. D. Working capital.

economic resources.

The measurement focus of the government-wide statements, proprietary funds, and fiduciary funds is on _____________________________.

Answer (A) is correct. An entity must apply the guidance on revenue from contracts with customers to all contracts with customers except for (1) lease contracts, (2) insurance contracts, (3) financial instruments, (4) guarantees (other than product or service warranties), and (5) nonmonetary exchanges between entities in the same line of business to facilitate sales to customers. Accordingly, product or service warranties are within the scope of the new standard.

The scope of the FASB's standard on revenue from contracts with customers (ASC 606) includes A. Product or service warranty contracts. B. Insurance contracts. C. Lease contracts. D. Loan guarantee contracts.

Answer (B) is correct. Interfund activity may be reciprocal or nonreciprocal. Reciprocal interfund activity is similar to exchange and exchange-like transactions, for example, interfund loans and services provided and used. Nonreciprocal interfund activity is similar to nonexchange transactions, for example, interfund transfers and reimbursements.

The standards for reporting interfund activity classifies such activity as A. Operating transfers and residual equity transfers. B. Reciprocal and nonreciprocal. C. Quasi-external transfers and residual equity transfers. D. Operating transfers, residual equity transfers, and reimbursements.

Answer (C) is correct. The statement of cash flows reports the cash effects of transactions. The accrual-basis gain on the stock is not relevant.

Three years ago, Jameson Company purchased stock in Zebra, Inc., at a cost of $100,000. This stock was sold for $150,000 during the current fiscal year. The result of this transaction should be shown in the investing activities section of Jameson's statement of cash flows as A. $50,000 B. $100,000 C. $150,000 D. Zero.

Answer (D) is correct. The statement of revenues, expenses, and changes in fund net position is the operating statement for proprietary funds (enterprise funds and internal service funds), which use the accrual basis of accounting. This statement distinguishes operating and nonoperating revenues and expenses. Internal service funds may be used for activities that provide goods and services to other subunits of the primary government and its component units or to other governments on a cost-reimbursement basis. The provision by an internal service fund of data processing services to other units of the same reporting entity is an interfund service provided and used. These services result in revenues to seller funds. *Given that the principal purpose of the internal service fund is to provide data processing services to other subunits of the primary government, the revenues are classified as operating.*

To report the billing for data processing services provided to Lud's other governmental units in the statement of revenues, expenses, and changes in fund net position, which of the following should be credited in the internal service fund? A. Interfund reimbursements. B. Interfund transfers. C. Data processing departmental expenses. D. Operating revenues.

Answer (A) is correct. Treasury stock is reported on the statement of financial position as a decrease in shareholders' equity.

Treasury stock is recorded on the statement of financial position as a(n) A. Decrease in shareholders' equity. B. Increase in shareholders' equity. C. Increase in assets. D. Decrease in assets.

1. Translation Method 2. Remeasurement Method

Two methods for converting financial statements from foreign to US currency:

An entity may choose either the cost model (as under U.S. GAAP) or the revaluation model as its accounting policy. It must apply that policy to an entire class of PPE. A class is a grouping of assets of similar nature and use in an entity's operations, for example, land, office equipment, or motor vehicles. Under the revaluation model, an item of PPE whose fair value can be reliably measured may be carried at a revalued amount equal to fair value at the revaluation date (minus subsequent accumulated depreciation and impairment losses).

Under IFRS an item of property, plant, and equipment must be carried at

Answer (D) is correct. An intangible asset is carried at cost minus any accumulated amortization and impairment losses, or at a revalued amount. The revaluation model is similar to that for items of PPE (initial recognition of an asset at cost). However, fair value must be determined based on an active market.

Under IFRS, an entity that acquires an intangible asset may use the revaluation model for subsequent measurement only if A. The cost of the intangible asset can be measured reliably. B. The useful life of the intangible asset can be reliably determined. C. The intangible asset is a monetary asset. D. An active market exists for the intangible asset.

Answer (B) is correct. Assuming an entity elects the revaluation model instead of the cost model, an item of PPE whose fair value can be reliably measured may be carried at a revalued amount equal to fair value at the revaluation date (minus subsequent accumulated depreciation and impairment losses). Revaluation is needed whenever fair value and the asset's carrying amount differ materially. If an item of PPE is revalued, all of the items in its class (e.g., land, buildings, or office equipment) also should be revalued. Accumulated depreciation is restated proportionately or eliminated.

Under IFRS, an entity that has elected to use the revaluation model for property, plant, and equipment (PPE) may revalue PPE as often as A. 30 days. B. Anytime the determination is made. C. Annually. D. 90 days.

Answer (B) is correct. Any indication that an asset may be impaired requires the entity to estimate the recoverable amount. The recoverable amount is the greater of an asset's fair value minus cost to sell or its value in use. Value in use is the present value of estimated future cash flows expected to be derived from the use and disposition of an asset.

Under IFRS, an impairment loss recognized is equal to the excess of the carrying amount of an asset over its recoverable amount. The recoverable amount of an asset is A. The historical cost of an asset minus accumulated depreciation. B. The greater of an asset's fair value minus cost to sell or its value in use. C. The present value of the future cash flows expected to be derived from an asset. D. The estimated selling price minus the estimated costs of completion and selling the asset.

Answer (C) is correct. The test for impairment under IFRS is whether the carrying amount of the CGU (including goodwill allocated solely for the purpose of impairment testing) exceeds its recoverable amount. Any excess is an impairment loss. Thus, the test has one step.

Under IFRS, what is the appropriate test for impairment of a cash-generating unit with goodwill (CGU)? A. Goodwill at the CGU level exceeds the sum of the components of goodwill at the next lower organizational levels. B. The carrying amount of the CGU exceeds its fair value. C. The carrying amount of a CGU exceeds its recoverable amount. D. The carrying amount of the reporting unit exceeds its fair value.

Answer (C) is correct. The remeasurement of plan assets for the period is the return on plan assets minus interest income on plan assets. Remeasurements of plan assets are recognized in OCI and are never reclassified to profit or loss.

Under IFRS, which of the following statements is true regarding remeasurements of plan assets by an entity that sponsors a defined benefit pension plan? A. They are measured as the difference between the actual return on plan assets and the expected return on plan assets. B. They must be recognized immediately in profit or loss. C. They must be recognized in OCI and must not be reclassified to profit or loss in subsequent periods. D. They must be recognized in OCI and may be amortized to profit or loss in subsequent periods according to the corridor approach.

1. tested for impairment only when a triggering event occurs indicating that the fair value of the entity is below its carrying amount. 2. The entity has an option to perform a qualitative assessment to determine whether the quantitative impairment test is needed. 3. A one-step quantitative test then must be performed. Impairment loss = Entity's carrying amount - Entity's fair value. Impairment loss recognized must not exceed the carrying amount of goodwill

Under the accounting alternative, goodwill recognized must be amortized on a straight-line basis over 10 years. A private company that applies this accounting alternative must elect whether to test goodwill for impairment on an entity level or a reporting unit level. The goodwill impairment test under the accounting alternative is simpler than under the general guidance. The following is the goodwill impairment test on an entity or reporting unit level:

15; expensed as incurred.

Under the federal tax code, organization and start-up costs must be capitalized and amortized over a period of not less than _______ years. For financial accounting purposes, organization costs must be __________________________

Answer (D) is correct. A 2-for-1 stock split doubles the number of shares outstanding; retained earnings is not affected. Under a stock dividend, however, a portion of retained earnings is reclassified as common stock. Since dividends are restricted by the amount of available retained earnings, a stock dividend, but not a stock split, will impair the firm's ability to pay dividends in the future.

Underhall, Inc.'s common stock is currently selling for $108 per share. Underhall is planning a new stock issue in the near future and would like to stimulate interest in the company. The Board, however, does not want to distribute capital at this time. Therefore, Underhall is considering whether to offer a 2-for-1 common stock split or a 100% stock dividend on its common stock. The best reason for opting for the stock split is that A. The impact on earnings per share will not be as great. B. The par value per share will remain unchanged. C. It will not decrease shareholders' equity. D. It will not impair the company's ability to pay dividends in the future.

Answer (C) is correct. A 2-for-1 stock split doubles the number of shares outstanding; retained earnings is not affected. Under a stock dividend, however, a portion of retained earnings is reclassified as common stock. Since dividends are restricted by the amount of available retained earnings, a stock dividend, but not a stock split, will impair the firm's ability to pay dividends in the future.

Underhall, Inc.'s common stock is currently selling for $108 per share. Underhall is planning a new stock issue in the near future and would like to stimulate interest in the company. The Board, however, does not want to distribute capital at this time. Therefore, Underhall is considering whether to offer a 2-for-1 common stock split or a 100% stock dividend on its common stock. The best reason for opting for the stock split is that A. The par value per share will remain unchanged. B. The impact on earnings per share will not be as great. C. It will not impair the company's ability to pay dividends in the future. D. It will not decrease shareholders' equity.

Answer (D) is correct. Net position includes (1) net investment in capital assets, (2) restricted net position, and (3) unrestricted net position. Unrestricted net position includes items that may be internally committed or assigned. However, the designation is not reported on the financial statements.

Unrestricted net position in the government-wide statement of net position A. Always equals unassigned fund balance in the governmental funds financial statements. B. May include internal commitments or assignments reported on the financial statements. C. Cannot be a negative amount. D. May include internal commitments or assignments not reported on the financial statements.

Answer (D) is correct. To provide an overview of component units, discrete presentation of component unit data is required in the government-wide statements. Each major component unit should be reported in the basic statements by presentation (1) in a separate column in the government-wide statements, (2) in combining statements of major component units after the fund statements, or (3) of condensed statements (a statement of net position and a statement of activities) in the notes. However, major component unit information is not required for fiduciary component units.

Users of a government's financial statements should be able to distinguish between the primary government and its component units. Furthermore, an overview of the discretely presented component units should be provided. Accordingly, A. Major component unit information must be provided in the form of combining statements. B. The government-wide statements provide discrete presentation of component unit data, including data for fiduciary component units. C. Condensed financial statements for major component units must be presented in the notes to the basic statements. D. Information about each major component unit must be provided in the reporting entity's basic statements.

3

Usually, investments with original maturities of ____ months or less qualify as cash equivalents.

The amortization method should reflect the pattern in which the economic benefits of the intangible asset are consumed. If the pattern is not reliably determinable, the straight-line method is used. Consequently, annual amortization is $2,240 ($112,000 ÷ 50 years).

Wall Company bought a trademark from Black Corporation on January 1 for $112,000. An independent consultant retained by Wall estimated that the remaining useful life is 50 years. Its unamortized cost on Black's accounting records was $56,000. Wall decided to write off the trademark over the maximum period allowed. However, the pattern of consumption of the economic benefits of the trademark is not reliably determinable. How much should be amortized for the year ended December 31? A. $2,240 B. $1,400 C. $1,120 D. $2,800

Answer (D) is correct. An intangible asset with a finite useful life to the reporting entity is amortized over that useful life. Amortization is based on the pattern of consumption of economic benefits, if reliably determinable. Otherwise, the straight-line method must be used. Thus, this intangible asset is amortized over 10 years using the straight-line amortization method. The amortizable amount of an intangible asset equals the amount initially assigned minus the residual value. The residual value is the estimated fair value to the entity at the end of the asset's useful life minus disposal costs. This amount is zero unless (1) a third party has committed to purchase the asset or (2) it can be determined from an exchange transaction in an existing market for the asset that is expected to exist at the end of the useful life. Thus, the amortizable amount of the intangible asset is $40,000 ($50,000 historical cost - $10,000 residual value), and the annual amortization expense is $4,000 ($40,000 amortizable amount ÷ 10 years).

West Co. paid $50,000 for an intangible asset other than goodwill. Fair value of the asset is $55,000. West signed a contract to sell the asset for $10,000 in 10 years. What amount of amortization expense should West record each year? A. $5,500 B. $5,000 C. $4,500 D. $4,000

Answer (B) is correct. The consideration transferred in a business combination (acquisition costs) is calculated as the sum of the acquisition-date fair values of the assets transferred by the acquirer, the liabilities incurred by the acquirer to former owners of the acquiree, and the equity interests issued by the acquirer. Company headquarters relocation cost is not part of the consideration transferred to acquire control over East. This cost relates to the operations after the acquisition date. Accordingly, these costs must not be included in West's acquisition cost.

West, Inc., acquired 60% of East Co.'s outstanding common stock. West paid $800,000 to acquire the stock. West plans to relocate East's company headquarters, which is expected to cost between $100,000 and $300,000. The present value of the probability-adjusted relocation cost is $240,000. What is West's acquisition cost? A. $1,100,000 B. $800,000 C. $1,040,000 D. $900,000

we are reconciling COGS to cash paid for goods sold. 1. Determine purchases by *subtracting* the *decrease in inventory* 2. determine cash paid for goods sold by *subtracting* the *increase in trade accounts payable*

What amount should Flax report in its statement of cash flows for the year ended December 31, Year 6, for cash paid for goods sold?

DR cash CR bonds payable for the face amount DR or CR premium or discount

What is the first entry you make when you issue bonds

Answer (C) is correct. In a nonexchange transaction, a government either gives or receives value without directly receiving or giving equal value in return.

What is the major difference between an exchange transaction and a nonexchange transaction for governmental units? A. Purpose restrictions placed upon fund balances. B. Time requirements and whether the transaction is required by law. C. The relationship between the amount of value given and received. D. Whether resources acquired can be further exchanged.

1. Qualitative assessment. The entity may choose to assess whether qualitative factors indicate that it is more likely than not (probability > 50%) that the fair value of the reporting unit is less than its carrying amount 2. Quantitative test. If the qualitative assessment indicates that potential impairment exists, the following two-step quantitative test is performed -compares the fair value of the reporting unit with its carrying amount. -compares the implied fair value of reporting-unit goodwill with the carrying amount of that goodwill. An impairment loss is recognized for the excess of the carrying amount of reporting-unit goodwill over its implied fair value. This loss is nonreversible.

What two tests are performed to test for impairment?

C. Serial.

What type of bonds mature in installments? A. Variable rate. B. Debenture. C. Serial. D. Term.

Answer (C) is correct. Estimated revenues control is an anticipatory asset recognized by a debit upon the adoption of the budget. Revenues control is a nominal account in which revenues are credited when they meet the recognition criteria. At year end, both accounts are closed. Because no other entries are made to estimated revenues control, the closing entry credits the account for the initial amount.

When Rolan County adopted its budget for the year ending June 30, Year 1, $20 million was recorded for estimated revenues control. Actual revenues for the year ended June 30, Year 1, amounted to $17 million. In closing the budgetary accounts at June 30, Year 1, A. Estimated revenues control should be debited for $3 million. B. Revenues control should be credited for $20 million. C. Estimated revenues control should be credited for $20 million. D. Revenues control should be debited for $3 million.

Answer (A) is correct- Acquisition-date fair value of the net assets acquired. Goodwill is an asset reflecting the future economic benefits arising from other assets acquired in a business combination. The other assets are not individually identified and separately recognized. The acquirer may recognize goodwill at the acquisition date

When a business is acquired, the acquirer may recognize goodwill. This amount is the excess of the sum of the acquisition-date fair values (with some exceptions) of (1) the consideration transferred, (2) any noncontrolling interest in the acquiree, and (3) a previously held equity interest in the acquiree over the A. Acquisition-date fair value of the net assets acquired. B. Carrying amount of the net assets acquired. C. Fair value of the net tangible assets acquired. D. Carrying amount of the net tangible assets acquired.

gain: included in asset revaluation surplus, an equity account, like other comprehensive income; loss: accounted for as an other loss included in income from operations.

When a class of assets' fair value can be reliably measured, a corporation applying IFRS can elect to apply the revaluation model to the class of assets. When the carrying value of the assets differs materially from the fair value of the assets, a revaluation must occur, with any increase being ___________________________________ and a decrease being

Answer (C) is correct. Consolidated financial statements should reflect the economic activities of an entity measured without regard to the boundaries of the legal entity. Accounting information pertains to an entity, the boundaries of which are not necessarily those of the legal entity. For instance, a parent and subsidiary are legally separate but are treated as a single entity in consolidated statements.

When a parent-subsidiary relationship exists, consolidated financial statements are prepared in recognition of the accounting concept of A. Legal entity. B. Materiality. C. Economic entity. D. Reliability.

When you receive cash, the cash is the partial realization of the gain. You divide the cash about by the total consideration amount and then multiply the cash by that % to get the gain you have to realize Debit cash Debit new asset Credit old asset Credit gain (% of cash)

When do you recognize a gain when the exchange is not of commercial substance?

Goodwill is recognized only in a business combination. It is "an asset representing the future economic benefits arising from other assets acquired in a business combination that are not individually identified and separately recognized." Internally generated goodwill must not be recognized in the financial statements. Goodwill is not amortized. Instead, goodwill of a reporting unit is tested for impairment each year at the same time.

When is goodwill recognized and is it tested for impairment or amortized?

Answer (A) is correct. The issue price of a bond is based on the market interest rate and reflects its fair value. The proceeds received from the sale of a bond equal the sum of the present values of the face amount and the interest payment (if the bond is interest-bearing). When bonds are issued between interest payment dates, the buyer includes accrued interest in the purchase price.

When purchasing a bond, the present value of the bond's expected net future cash inflows discounted at the market rate of interest provides what information about the bond? A. Price. B. Par. C. Interest. D. Yield.

Answer (D) is correct. Goodwill can be recognized only in a business combination. Goodwill is an asset representing the future economic benefits arising from other assets acquired in a business combination that are not individually identified and separately recognized.

When should goodwill be recognized? A. The fair market value of the company's assets exceeds the book value of the company's assets. B. Costs have been incurred in the development of goodwill. C. The company expects a future benefit from the creation of goodwill. D. Goodwill has been created in the purchase of a business.

the asset has an indefinite life. The cash flows could continue for many years.

When testing for impairment, the undiscounted future net cash inflows are not considered when

You capitalize it with the land. It's the cost incurred to get the land in the condition for use.

When you buy land and destroy an old building on it so you can make a new one how is the cost treated?

Answer (D) is correct. Individual items or collections of works of art, historical treasures, and similar assets ordinarily must be capitalized. However, if a collection is (1) held in furtherance of public service and not for gain; (2) protected, preserved, cared for, and kept unencumbered; and (3) subject to a policy that sale proceeds are to be used to obtain other collection items, capitalization is not required. If capitalized collections or individual items are exhaustible, for example, because their useful lives are reduced by display, educational, or research uses, they must be depreciated.

Which capital assets must be depreciated in the government-wide financial statements? A. All infrastructure assets. B. All noncapitalized collections of historical treasures. C. All capitalized collections of works of art. D. All capitalized collections that are exhaustible.

Answer (B) is correct. An enterprise fund is a type of proprietary fund. Proprietary funds are required to present a statement of cash flows using the direct method to report operating cash flows.

Which format must an enterprise fund use to report cash flow operating activities in the statement of cash flows? A. Indirect method, beginning with operating income. B. Direct method. C. Indirect method, beginning with change in net position. D. Either direct or indirect method.

Answer (D) is correct. A statement of cash flows is required as part of a full set of financial statements of all business and not-for-profit entities. The primary purpose of a statement of cash flows is to provide information about the cash receipts and payments of an entity during a period. A secondary purpose is to provide information about operating, investing, and financing activities. The financing activities section of a cash flow statement would clearly show if the company has cash inflows from the sale of debt or equity securities.

Which is the most appropriate financial statement to use to determine if a company obtained financing during a year by issuing debt or equity securities? A. Balance sheet. B. Income statement. C. Statement of changes in equity. D. Statement of cash flows.

Answer (A) is correct. Fiduciary funds account for resources held by the government in trust or as an agent for (1) specific individuals, (2) private organizations, or (3) other governments. These resources are not available to finance the government's programs. Consequently, they are reported only in the fund statements (statements of fiduciary net position and changes in fiduciary net position).

Which of the following activities should be excluded when governmental fund financial statements are converted to government-wide financial statements? A. Fiduciary activities. B. Government activities. C. Proprietary activities. D. Enterprise activities.

Answer (A) is correct. If an entity that presents a full set of financial statements has items of other comprehensive income (OCI), it must present comprehensive income either (1) in a single continuous statement of comprehensive income or (2) in two separate but consecutive statements (an income statement and a statement of OCI).

Which of the following are acceptable formats for reporting comprehensive income? In one continuous financial statement In a statement of changes in equity In a separate statement of net income In two separate but consecutive financial statements A. I and IV only. B. I, II, and III only. C. III and IV only. D. I and II only.

Answer (A) is correct. Patents may be purchased or developed internally. The initial capitalized cost of a purchased patent is normally the fair value of the consideration given, that is, its purchase price plus incidental costs, such as registration and attorney's fees. Internally developed patents are less likely to be capitalized because related R&D costs must be expensed when incurred. Thus, only relatively minor costs can be capitalized, for example, patent registration fees and legal fees.

Which of the following expenditures qualifies for asset capitalization? A. Legal costs associated with obtaining a patent on a new product. B. Cost of materials used in prototype testing. C. Costs of testing a prototype and modifying its design. D. Salaries of engineering staff developing a new product.

Answer (C) is correct. Mandatorily redeemable financial instruments (MRFIs) are redeemable shares that embody an unconditional obligation to transfer assets at a fixed or determinable time or upon an event certain to occur. MRFIs must be accounted for as liabilities.

Which of the following financial instruments issued by a public company should be reported on the issuer's books as a liability on the date of issuance? A. Cumulative preferred stock. B. Preferred stock that is convertible to common stock 5 years from the issue date. C. Common stock that contains an unconditional redemption feature. D. Common stock that is issued at a 5% discount as part of an employee share purchase plan.

Answer (D) is correct. Governmental units normally record the purchases of supplies inventory in the general fund. In accounting for supplies, the expenditure account may be debited when the materials and supplies are purchased or when they are consumed. Under either method, the inventory of supplies remaining at year-end must be recorded on the balance sheet as an asset. In addition, under the purchases method, resources already have been expended to acquire these supplies. Accordingly, an appropriate amount of fund balance-nonexpendable must be credited to indicate the unavailability of resources in this amount for other expenditures.

Which of the following fund types used by a government most likely would have a fund balance classification used for an inventory of supplies? A. Capital projects. B. Internal service. C. Private-purpose trust. D. General.

Answer (A) is correct. Government-wide financial statements do not display funds or fund types but instead report information about the government as a whole. Two distinctions are made in the government-wide financial statements: (1) between the primary government and its discretely presented component units and (2), within the primary government, between governmental activities and business-type activities. Governmental activities are normally financed by nonexchange revenues (taxes, etc.). They are reported in governmental (for example, debt service) and internal service funds. Business-type activities are financed at least in part by fees charged to external parties for goods and services. They are usually reported in enterprise funds.

Which of the following funds should be reported as part of a local government's governmental activities column in its government-wide statements? A. Debt service. B. Private-purpose trust. C. Agency. D. Pension trust.

Answer (A) is correct. An intangible asset with an indefinite useful life other than goodwill must be reviewed for impairment at least annually. A qualitative assessment may be performed prior to determining whether it is necessary to perform the quantitative impairment test. If the entity cannot determine based on quantitative factors that it is more likely than not that an indefinite-lived intangible asset is not impaired, a quantitative impairment test must be performed. In a quantitative test, the carrying amount of an asset is compared with its fair value. If the carrying amount exceeds the fair value, the asset is impaired and the excess is the recognized loss.

Which of the following is a pair of values that are compared to determine the amount of a possible impairment loss on an intangible asset, with an indefinite life, other than goodwill? A. Fair value, carrying value. B. Fair value, present value. C. Carrying value, book value. D. Future value, carrying value.

Answer (B) is correct. A sponsoring governmental entity should report the external portion of each of its external investment pools as an investment trust fund. Separate statements of fiduciary net position and changes in fiduciary net position should be presented for each such fund. The external portion belongs to legally separate entities not included in the reporting entity.

Which of the following is a required financial statement for an investment trust fund? A. Statement of revenues, expenses, and changes in fiduciary net position. B. Statement of changes in fiduciary net position. C. Statement of activities. D. Statement of revenues, expenditures, and changes in fiduciary net position.

D. Development or improvement of techniques and processes.

Which of the following is a research and development cost? A. Market research related to a major product for the company. B. Research and development performed under contract for others. C. Offshore oil exploration that is the primary activity of a company. D. Development or improvement of techniques and processes.

Answer (C) is correct. Intangible assets include those with finite useful lives (an amortized intangible asset) and those with indefinite useful lives (a nonamortized intangible asset). An intangible asset with a finite useful life (e.g., a patent) is reviewed for impairment when events or changes in circumstances indicate that its carrying amount may not be recovered. The test for impairment is a two-step test: (1) perform the recoverability test (the carrying amount is not recoverable if it exceeds the sum of the undiscounted future cash flows expected from the use and disposition of the asset) and (2) determine the impairment loss for the excess of the carrying amount over the fair value. A patent is an intangible asset with a finite useful life, so it is subject to the recoverability test when testing for impairment.

Which of the following is an intangible asset that is subject to the recoverability test when testing for impairment? A. A trademark with indefinite useful life. B. Goodwill. C. A patent. D. R&D costs for a patent.

Answer (C) is correct. Seasonal or other periodic design changes to existing products are activities not classified as R&D.

Which of the following is not classified as R&D? A. Construction of a preproduction prototype. B. Evaluation of process alternatives. C. Periodic design changes to existing products. D. Design of tools involving new technology.

Answer (D) is correct. The statement of revenues, expenses, and changes in fund net position of proprietary funds reports (1) operating revenues, expenses, and income (loss); (2) nonoperating revenues and expenses; (3) revenues from capital contributions and additions to endowments; (4) special and extraordinary items; (5) transfers; (6) change in net position; and (7) ending net position.

Which of the following is presented on the proprietary funds statement of revenues, expenses, and changes in fund net position of a state or local government? A. Unrestricted net position. B. Assets and liabilities. C. Expenditures. D. Transfers.

Answer (C) is correct. A hedge is used to avoid or reduce risks by creating a relationship by which losses on certain positions are expected to be counterbalanced in whole or in part by gains on separate positions in another market. A perfect hedge is completely effective. It has a complete negative correlation with the item being hedged and results in no net gain or loss.

Which of the following is the characteristic of a perfect hedge? A. No possibility of future gain only. B. No possibility of future loss only. C. No possibility of future gain or loss. D. The possibility of future gain and no future loss

Neither. Short-term liabilities, such as accounts payable, do not usually provide for a periodic payment of interest unless the accounts are not settled when due or payable. They also are usually not secured by collateral.

Which of the following is usually associated with payables classified as accounts payable? 1. Periodic Payment of Interest 2. Secured by Collateral

Answer (B) is correct. An enterprise fund is a type of proprietary fund. All funds not governmental funds use the accrual basis of accounting.

Which of the following local government funds uses the accrual basis of accounting? A. Special revenue. B. Enterprise. C. Capital projects. D. Debt service.

Answer (B) is correct. A material contingent loss must be accrued (debit loss and credit contingent liability) when (1) it is probable that, at a balance sheet date, an asset has been impaired or a liability has been incurred and (2) the amount of the loss can be reasonably estimated. If the amount of loss cannot be reasonably estimated but the probability of the loss is at least reasonably possible, the nature of the contingency must be described in the notes to the financial statements.

Which of the following methods should a company use to account for a contingent liability when the loss is probable but not reasonably estimated? A. The liability should not be reported. B. The liability should only be disclosed in the notes to the financial statements. C. The liability should be reported as a long-term liability. D. The liability should be reported as a short-term liability.

Answer (B) is correct. Providing information for assessing service efforts and accomplishments is one of the objectives that serves to assist in evaluating the operating results of a governmental entity.

Which of the following objectives of financial reporting is applicable to governmental entities? Provide information A. Useful in assessing management stewardship and performance. B. For assessing service efforts and accomplishments. C. About economic resources, obligations, net resources, and changes in them. D. Useful in making resource allocation decisions.

Answer (B) is correct. The objectives of accounting for income taxes are to recognize (1) the amount of taxes currently payable or refundable and (2) the deferred tax liabilities and assets for the future tax consequences of events that have been recognized in the financial statements or tax returns.

Which of the following statements is a primary objective of accounting for income taxes? A. To compare an enterprise's federal tax liability to its state tax liability. B. To recognize the amount of deferred tax liabilities and deferred tax assets reported for future tax consequences. C. To identify all of the permanent and temporary differences of an enterprise. D. To estimate the effect of the tax consequences of future events.

Answer (B) is correct. Start-up costs are expenses incurred to begin a business activity, e.g., costs of organization, opening a facility, or product introduction. Organization costs are those incurred in the formation of a business entity. Under the federal tax code, organization and start-up costs must be capitalized and amortized over a period of not less than 15 years. However, for financial accounting purposes, nongovernmental entities must expense all start-up and organization costs as incurred.

Which of the following statements is correct concerning start-up costs? A. Costs of start-up activities, including organization costs, should be capitalized and amortized on a straight-line basis over the lesser of the estimated economic life of the company or 60 months. B. Costs of start-up activities, including organization costs, should be expensed as incurred. C. Costs of start-up activities should be capitalized and amortized on a straight-line basis over the lesser of the estimated economic life of the company or 60 months, while organization cost should be expensed as incurred. D. Costs of start-up activities, including organization costs, should be capitalized and expensed only if an impairment exists.

Answer (A) is correct. The modified accrual basis of accounting is used to report the governmental fund financial statements. The measurement focus is on current financial resources, that is, on determining financial position and changes in it. Thus, an amount to be paid from current financial resources should be reported as an expenditure (a decrease in current financial resources) and a liability when the liability is incurred (i.e., when goods or services are acquired).

Which of the following transactions should be reported as a liability in the general fund financial statements? A. An amount to be paid from current financial resources. B. An amount set aside to pay for an unfilled contract. C. An amount that is due within one year of the balance sheet date. D. Principal on long-term debt due 90 days after the balance sheet date.

Answer (B) is correct. Internally developed intangibles other than goodwill are most often initially recorded at the amount of the incidental costs only. Thus, cost of patent registration is capitalized as patent cost and reported in the financial statements as an intangible asset.

Which of the following types of assets would typically be reported on a company's balance sheet as an intangible asset? A. Derivative securities. B. Cost of patent registrations. C. Leasehold improvements. D. Cost of research and development.

Answer (D) is correct. The statement of activities presents operations in a format that displays net (expense) revenue for each function. The purpose is to report the relative financial burden to the taxpayers for that function. The net (expense) revenue for each governmental or business-type function equals expenses (at a minimum, the direct expenses of the function) minus program revenues. Charges for services are program revenues resulting from charges to customers, applicants, or others who directly benefit from what is provided (goods, services, or privileges) or who are otherwise directly affected.

Which of the following would be reported as program revenues on a local government's government-wide statement of activities? A. Taxes levied for a specific function. B. Interest revenues. C. Proceeds from the sale of a capital asset used for a specific function. D. Charges for services.

Answer (A) is correct. Operating activities are all transactions and other events that are not financing or investing activities. In general, operating activities involve the production and delivery of goods and the provision of services. Their effects normally are reported in earnings. A decrease in accounts payable indicates a cash outflow to the entity's suppliers in payment for goods or services.

Which one of the following should be classified as a cash flow from an operating activity on the statement of cash flows? A. A decrease in accounts payable during the year. B. An increase in cash resulting from the issuance of previously authorized common stock. C. The payment of a cash dividend from money arising from current operations. D. The payment of cash for the purchase of additional equipment needed for current production.

A private company. Under the accounting alternative, goodwill recognized must be amortized on a straight-line basis over 10 years. The amortization expense is recognized in the income statement.

Who can Elect to apply the goodwill accounting alternative and what is it?

Answer (B) is correct. Organization costs are those incurred in the formation of a business entity. Under the federal tax code, organization and start-up costs must be capitalized and amortized over a period of not less than 15 years. However, for financial accounting purposes, nongovernmental entities must expense all start-up and organization costs as incurred.

Wind Co. incurred organization costs of $6,000 at the beginning of its first year of operations. How should Wind treat the organization costs in its financial statements in accordance with GAAP? A. Amortized over 40 years. B. Expensed immediately. C. Never amortized. D. Amortized over 180 months.

Answer (B) is correct. The costs of equipment acquired or constructed for a particular project and having no alternative future uses and therefore no separate economic values are R&D costs and are expensed when incurred. The costs of equipment acquired for R&D and having alternative future uses are capitalized as tangible assets when acquired or constructed. Thus, Machine A should be expensed and Machine B should be capitalized. The cost to include in R&D relating to Machine A is $250,000, the entire cost of the machine. The cost to be included in R&D relating to Machine B is the straight-line depreciation of $25,000 ($250,000 ÷ 10). Total R&D expense for the machines used is $275,000 ($250,000 + $25,000).

Wizard Co. purchased two machines for $250,000 each on January 2. The machines were put into use immediately. Machine A has a useful life of 5 years and can be used in only one research project. Machine B will be used for 2 years on a research and development project and then used by the production division for an additional 8 years. Wizard uses the straight-line method of depreciation. What amount should Wizard include in research and development expense for the year? A. $75,000 B. $275,000 C. $500,000 D. $375,000

Discount; discount

___________ amortized, interest expense, and the carrying amount of the bonds increase each period when amortizing a ____________.

Revenue; revenue

____________ bonds are issued by governments and are payable from specific ______________ sources.

Goodwill

__________________ is tested for impairment at least annually but is never amortized.

Debentures

___________________ are backed by the borrower's general credit and not by specific collateral.

Research; Development (Market research or testing is excluded from Development)

_____________________ is planned search or critical investigation aimed at discovery of new knowledge with the hope that it will be useful in developing a new (or significantly improving an existing) product, service, process, or technique (product or process). __________________ is translation of research findings or other knowledge into a plan or design for a new or improved product or process.

1. it's part of a collection 2. the collection is held for the public's benefit 3. if an item of the collection is sold, the proceeds are used to purchase additional collection items

a NFP doesn't have to record a gift of art/antiques IF

increases

amortization of a bond discount always ____________ CV

decreases

amortization of a bond premium always ____________ CV

it requires a journal entry because the government uses an encumbrance system. DR. encumbrances control CR budgetary fund balance- reserved for encumbrances

another major difference in modified accrual accounting: sending out purchase orders

1. record the goodwill- if they bought 80% of the stock, divide the price by 80% to see what 100% is and then add the difference in FV to get the goodwill. 2. write the sub's asset to FV 3. eliminate the sub's capital structure (apic, RE, etc.)

consolidations: when the parent purchases more than 50% of the sub, they have to present consolidated financials. What are the three steps to start eliminating the the purchase/investment account?

the avg MP. if the MP is higher than that exercise price, there will be. if it's below, it's anti-dilutive. ignore those financial instruments then.

dilutive EPS: if you're using the treasury method to figure out the number of shares of warrants/options that are possibly dilutive, what will tell you that there will be dilutive shares? what tells you there absolutely won't?

PV of minimum lease payments.

do you capitalize a leased asset at the FV or the PV of the minimum lease payments?

available and measurable

governments accrue tax revenue when it's

DR est. revenues control 100,000 CR appropriations control 97,000 Other balancing accounts are added depending on what is on the budget

governments make journal entries to record their budgets. what entry do they make when they have estimated revenue and are approved to spend only a certain amount (legislative authority)?

always a change in estimate and principle, so that means you treat it as a change in estimate and adjust things prospectively.

how do you account for a change in depreciation method?

Research and development (R&D) costs must be expensed as incurred. Under IFRS, research costs must be expensed as incurred. Development costs may result in recognition of an intangible asset

how do you account for research and development fees? how about under IFRS?

4(4+1) / 2 = 10 you can also just add 1+2+3+4 = 10 (4/10) * 700,000 = 280,000 year two: (3/10) * 700,000 = 126,000 each year is calculated on the cost - salvage value only.

how do you calculate depreciation under sum of the year's digits when useful life = 4 years building = $800,000 salvage = $100,000

you back out bed debt expense (charity service to homeless) and also 3rd party contractual adjustments (insurance only pays $800 for a scan but hospital bills $1000)

how do you calculate net patient service revenue for a private, NFP hospital?

over the life of the *asset* if it a transfer of ownership or bargain purchase option over the life of the *lease* if it is 75% life or 90% FV

how do you depreciate the asset when it's classified as a capital lease?

it's your replacement cost. however it cannot fall below the floor or be above the ceiling. It's the middle number when you look at all three.

how do you get your market price?

Sales revenue is recognized only for the amount of consideration to which an entity expects to be entitled. Thus, no sales are recognized for the products expected to be returned. A refund liability is recognized for the amount of consideration expected to be returned to customers. The adjustments to the refund liability are recognized as revenue (or reductions of revenue).

how do you record sales that are likely to be returned?

temporary- Due from/due to permanent- other financing source/other financing uses

how do you record transfers of money between governmental funds?

Current exchange rate: all monetary items on the BS and then dividends (date of declaration) Historical exchange rate: all nonmonetary items on the BS, depreciation, COGS, common stock, and APIC Weighted-average exchange rate: revenues and expenses

how do you use the re-measurement method when converting a subs financials to US dollars?

current exchange rate (BS date): all assets and all liabilities Weighted average method: revenues and expenses

how do you use the translation method when converting a subs financials to US dollars?

lower of cost or market lover of cost or NRV

how do you value inventory under LIFO/retail? FIFO?

the close out the income statement revenue account and credit a net assets temporarily constricted account

how does a NPF record restricted revenue from a grant at year-end that they haven't spent yet?

because of temporary differences, you have two entries. 1. what you owe the government DR income tax expense CR income tax payable 2. record the deferred portion DR deferred tax asset DR deferred tax expense (PLUG) CR deferred tax liability or . DR deferred tax asset CR deferred tax benefit (PLUG) CR deferred tax liability

how many entries does it take to record a corporations taxes?

at least annually and when events or changes in circumstances indicate that its carrying amount may not be recoverable.

how often do you test an intangible asset with an indefinite life for impairment?

DR expenditures control CR vouchers payable treat the purchase of a fixed asset as an expense

if a governmental fund goes out and buys a fixed asset, what entry can they make?

yes. both.

if a stock spit or dividend takes place after year end but before the financials are issued, should they be taken into consideration for the EPS calculation?

if ending inventory is understated, then COGS will be overstated and RE understated

if ending inventory is understated, what happens to COGS and ending RE?

DR uncollectible tax to make it what amount was actually uncollectible. CR revenues control because you didn't record enough revenue originally.

if you collect more taxes than you originally estimated, you'll DR cash and CR taxes rec, but then you'll have too large of a credit in taxes receivable, so what adjustment will you have to make?

the lessor's rate if it -is lower than that lessee's rate -is known to the lessee otherwise use the lessee's rate

if you get two interest rates when you're doing a capital lease problem, which rate do you use?

an outsider/3rd party

in NPF accounting, the only way money can be restricted is by

1. property taxes 2. real estate taxes 3. income taxes 4. sales taxes 5. tax payments due from other governmental bodies

in modified accrual, there are only 5 types of revenue that you'll accrue

FIFO

in periods of inflation when is inventory valued the same under the periodic method and perpetual method? FIFO or LIFO?

1. specific purpose fund (temporarily restricted) 2. plant replacement and expansion fund (temporarily restricted) 3. endowment funds- -permanent/pure endowment (can never spend the principal) -term endowment (after a period of time you can spend the principal)

internal reporting- where do the *donor restricted* funds go in a NPF (four accounts)

inventory turnover = COGS / Avg Inventory

inventory turnover ratio

goodwill is not amortized. it is tested for impairment each year at the same time. cost of maintaining the goodwill should be expensed.

is goodwill amortized?

debit cash credit sales credit refund liability debit COGS debit return asset credit inventory

journal entry to record sales but with the contract being variable (refunds are probable)

1. patient service revenue- includes charity care, bad debts, and 3rd party contractual adjustments that you back-out for net 2. other operating revenue- CPR classes, gift shop, cafateria 3. non-operating revenue- gain/loss from sale or asset, donations (unrestricted)

nonprofit hospitals turn the statement of activities into two statements: 1. statement of operations 2. statement of changes in net assets but they also divide their revenue under the statement of operations into three categories:

transactions between funds- not transfers. the fund that sells: DR Cash CR Revenue control the fund that receives: DR expenditures control CR cash

quasi-external transaction

permanent transfer from one fund to another but not for operating purposes. it gets an enterprise fund started or internal service fund. if the fund is a proprietary fund, they have an account called transfers in. CR transfers in

residual equity transfer

you have to defer the gain up to the PV of the lease payments. look at like more than 10% or less than 90%

sale-leaseback: do you defer the gain if you have less than a major interest but more than a minor interest in the asset?

1. is the term of the lease a large % of the remaining life of the asset? or 2. The PV of the lease payments greater than or equal to 90% of the FMV of the asset

sale-leaseback: how do you know if you should defer the gain? (disregard less than a major but more than a minor)

yes if they meet the criteria: 1. the FMV of the services can be determined AND 2. either the donated skills would have had to be paid for otherwise or nonfinancial assets have been created or enhanced

should donated services be recorded?

solvency- a business' ability to pay it's *noncurrent* debts as they become due and thus remain in business in the long run. liquidity- a business' ability to pay it's *current" debt as it becomes due

solvency versus liquidity

DR Retained earnings 2,000,000 = $20 X 1,000,000 X 10% CR Common stock 100,000 = $1 X 1,000,000 X 10% CR APIC 1,900,000 PLUG stock dividends or splits never affect the total amount of equity. you are just reorganizing it- capitalizing.

stock dividend journal entry for issuing a -10% stock dividend -1,000,000 shares outstanding -$1 Par value -$20 Market Value

1. memorandum entry- not journal entry, just note that par decreased and # of shares increased. 2. capitalize RE at the par value of shares issued. DR Retained Earnings at par CR Common Stock at par

stock split journal entry

Other financing sources (if they receive the money) Other financing uses (if they are sending the money)

the exam loves transfers between the five governmental funds. What do they call these accounts for a *permanent* transfer?

earnings before interest and taxes divided by interest expense. The times-interest-earned ratio is a measure of the firm's ability to pay interest on debt.

times-interest-earned ratio

the lower of cost or net realizable value (NRV)

under IFRS Inventories are measured at ____________________________________________________ regardless of the cost method used.

it's like maintenance. DR executory cost CR cash usually do it with the first payment.

what are annual executory costs in a lease?

On the face of the balance sheet, the bonds payable are reported at Bonds Payable = face amount - bond discount + bond premium - debt issue costs

what are bonds payable reduced or increased by when reported on the BS?

1. Enhancing 2. Comparability 3. Verifiability 4. Timeliness 5. Understandability (every coach values team unity)

what are the components that enhance the qualitative characteristics of financial information?

1. TO- The lease provides for the transfer of ownership. 2. BPO- The lease contains a bargain purchase option. 3. The lease term is 75% or more of the estimated economic life of the leased property. 4. The present value of the minimum lease payments is at least 90% of the fair value of the leased property to the lessor at the inception of the lease.

what are the criteria to classify as a lease as capital?

1. pension trust fund 2. private purpose trust fund 3. investment trust fund 4. agency fund

what are the four funds in the fiduciary fund

1. *Pension (and other employee benefit) trust funds* account for employee benefit programs. 2. *Investment trust funds* account for resources held for investment on behalf of other governments in an investment pool. 3. *Private-purpose trust funds* account for all other trust arrangements that benefit individuals, private organizations, or other governments. 4. *Agency funds* account for resources held temporarily in a purely custodial capacity, such as tolls that will be paid to a private business.

what are the four funds under the fiduciary fund?

1. *the unexpended plant fund*- accumulates the money to acquire capital additions (gifts/donations) (temporarily restricted) 2. *the investment in plant fund*- carries the fixed assets and any long-term debt secured by the FAs. (unrestricted- once you acquire a FA, there are no restrictions most times) 3. *the retirement of indebtedness plant fund*- service debt (net assets temporarily restricted) 4. *renewal and replacements plant fund*- major repairs, maintenance, refurbishing (net assets temporarily restricted)

what are the plant funds in a private, not-for-profit college

1. governmental funds 2. proprietary funds 3. fiduciary funds

what are the three broad funds that a governmental unit uses

1. *cash flows from operating activities*- (income statement) revenues, expenses, trading securities, interest I/E, dividends received 2. *cash flows from investing activities*- buy/sell HAPPEE. held-to-maturity, available-for-sale, property, plant, equipment, equity. 3. cash flows from financing activities- debt principal and dividends PAID, stock, treasury stock.

what are the three sections of the statement of cash flows:

bonds payable long term notes payable you have to credit an account called *other financing sources*

what are the two accounts you would never see in the four governmental funds

1. enterprise funds 2. internal service funds

what are the two funds under the tier 1 proprietary fund?

authorized and issued

what are treasury shares considered? -authorized -issued -outstanding

1. stock warrants/options 2. convertible preferred stock 3. convertible bonds 4. contingent shares

what can be dilutive to EPS?

-the measurement focus- the flow of current financial resources. -they all use modified accrual accounting

what do ALL 5 governmental funds have in common?

1. general fund 2. special revenue fund 3. capital fund 4. permanent fund -none can carry their own long-term debt- you would never see the account *bonds payable* or *long term notes payable* -not allowed to service their long term debt. you would never see *interest expense*

what do four of the governmental funds have in common?

DR APIC CR dividends payable does not effect RE.

what do they mean when they say they paid a liquidating dividend?

the numerator is net income available to common stock holders, so you back out any dividends owed to preferred shareholders. but don't back out what was paid in that year, just what was owed for the current year. they could tell you they paid $16,000 but par*% is only $10,000 and they paid more because of dividends in arrears. back 10,000 out of NI.

what do you back out of the numerator for EPS?

the lessee adds it to the asset's value that it will depreciate.

what do you do with finders fee that a lessee ?

it's twice the straight-line rate but you DO NOT back out salvage value. 800,000 building 100,000 salvage value 8 yr life 1/8 years = 12.5% 12.5% * 2 = 25% 25% * 800,000 = 200,000 depreciation yr 1 same thing with year 2! 800,000 - 200,000 = 600,000 600,000 * 25% = 150,000 DON'T BACK OUT SALVAGE BUT CALCULATE ON NEW CARRYING VALUE AFTER YOU BACKOUT EACH YEAR'S DEPRECIATION

what do you do with salvage value under the double-declining method of depreciation?

Costs to issue debt securities must be reported in the balance sheet as a direct deduction from the face amount of the debt. Debt issue costs should be amortized over the term of the debt using the interest method. But the straight-line amortization method may be applied if the results are not materially different.

what do you do with the cost of issuing a bond?

payment per period × future value factor of an ordinary annuity × (1 + %)= FV of an annuity due

what do you have to do differently to get the FV of an annuity due?

payment per period × present value factor of an ordinary annuity × (1 + %)= PY of an annuity due

what do you have to do differently to get the PV of an annuity DUE?

-change in net assets- unrestricted -change in net assets- temporarily restricted -change in net assets- permanently restricted

what does a NFP balance sheet show

it puts a ceiling on what common stockholders receive. so you do all the preferred dividends and dividends in arrears, then you have to give the common stockholders the rest, but since the preferred shareholders are participating, the common stockholders can only get up to the same percent as the preferred. the extra participation amount is then a ratio between preferred and common shareholders based on the total shares.

what does it mean if preferred stock is participating?

they are issued for less than face value because they want the bond to yield more than the stated rate.

what does it mean when a bond is issued at a discount?

it prohibits stock dividends from being paid on treasury stock- if they are just talking about a 2 for 1 split, ignore it. it only protects agains stock dividends.

what does it mean when they say that the state protects the treasury stock from dilution?

you can only spend the dividends and interest on a specified operation. you cannot spend the principle

what does it mean when we say that the governmental permanent fund is NOT expendable

you can spend every bit of dividends, interest, and principle on the activity that the fund is used for

what does it mean when you say that the governmental special revenue fund is expendable?

they preferred to give up their voting rights for a preference to dividends

what does preferred dividends mean

it lowers you ending inventory. You're going from putting the lower priced pieces of inventory on the income statement to putting the higher priced (most recently purchased) items on the income statement. So therefore you inventory is going down much faster.

what does switching from FIFO to LIFO do to the balance sheet?

nothing. they record a memorandum entry and racalc their cost per share.

what does the investor record when they receive a stock dividend or split?

They only debit purchases and credit AP. They do nothing to COGS until they do an inventory count where they see what's left in inventory. COGS = beg + purchases - ending

what entries does a purchaser make under the periodic accounting for inventory?

They don't use a purchases account. The debit inventory and credit AP. Then when a sale is made they are accounting for COGS right away. They then debit AR credit sales credit inventory debit COGS You don't go and count inventory periodically to see what your ending inventory is and to calc COGS. you do though need to do a physical count of inventory to see if there's any spoilage or theft.

what entries does a purchaser make under the perpetual accounting for inventory?

DR cash CR revenues control

what entry do you make when you collect meter money?

DR revenue CR cash

what entry does a NFP college make for refunds of tuition?

cash = $150 tickets FMV = $100 DR cash 150 CR tickets inventory 100 CR contribution revenue

what entry does a nonprofit make when they receive a donation of money (like a raffle) and the donor then receives tickets from them?

when a company has other securities outstanding that have the potential to be common stock. not just if they have preferred stock- it has to be convertible. DILUTIVE EPS

what is a complex capital structure regarding earnings per share?

noncurrent asset

what is a deferred tax liability classified as?

when the government is acting like an agent for someone else. an example would be collecting property taxes for a bunch of cities. you DR cash and CR each district (due to district 1). all you see are cash and due to xyz with this fund. you don't see revenues or expenditures. this is just a temporary holding account.

what is an agency fund under tier two?

a potential obligation. we keep track of potential obligations which is why you make a journal entry when you send out a purchase order.

what is an encumbrance?

interest expense = CV X Effective Rate interest payable = FV X Stated rate

what is interest expense on a bond equal to? what is interest payable on a bond equal to?

it's restricted by a 3rd party. outsider.

what is restricted funds?

the actual revenue you think you will receive/receive. a CR.

what is the account *revenues control* for?

Appropriations control

what is the account called in governmental fund accounting when there is a restriction (legislative authority) on how much of a tax revenue the fund is allowed to spend?

it's an IS account

what is the account net assets released from restriction in NPF accounting?

face amount plus any unamortized premium or minus and unamortized discount

what is the carrying value of a bond? This is what debt is reported at on the financials.

DR scholarship expenditures CR Revenue record the revenue as if people will pay the tuition, but it washes out with the expenditure. just disclosure

what is the entry a NFP college makes when they give out grants/scholarships/free tuition/tuition waver?

DR salaries/wage expense CR revenue disclosure. you book it as an ep

what is the entry when a NFP receives a donation of service

DR the amount in RE DR APIC (plug) CR Dividends payable for the entire amount

what is the entry you make when you declare a dividend greater than your accumulated earnings (RE)?

Fiduciary funds also provide information about operational accountability. They account for resources held by the government in trust or as an agent for (a) specific individuals, (b) private organizations, or (c) other governments. Accordingly, they cannot be used for the reporting entity's purposes.

what is the fiduciary fund in tier one?

effective yield X carrying value = interest expense DR interest expense- effective yield X carrying value CR interest payable- stated rate X face value The balancing entry is the discount/premium amotrization

what is the interest that is paid on a bond under the effective interest rate?

stated rate X face. everything on the bond.

what is the interest that is paid on a bond? INTEREST EXPENSE

none of the five funds are allowed to carry their own fixed assets if one of these funds buys a fixed asset, they can't debit machinery or equipment, they have to debit *expenditures control* and credit *vouchers payable* *treat a fixes asset purchase as an expense*

what is the major difference in modified accrual accounting for governmental funds on the *expenditure* side?

normal accrual accounting recognizes revenue as it's earned, but since everything is funded by tax revenue, this doesn't work. the governmental funds do not *earn* tax dollars. *governments accrue tax revenue when it's available and measurable* if a government levies taxes, it's not available and measurable. you have to estimate the expected collections within the year and 60 days after. that money is available and measurable

what is the major difference in modified accrual accounting for governmental funds on the *revenue* side?

Proprietary funds provide operational accountability information about the business-type activities of a government. Proprietary funds serve defined customer groups and generally are financed through fees. tier 2- enterprise funds and internal service funds

what is the proprietary fund in tier one?

board designated assets

what is the quasi-endowment fund in a NFP college?

contra-equity item

what is treasury stock on the BS?

treasury share method...has nothing really to do with treasury share. just take (avg. MP - exercise price) / avg MP then multiply that by the number of possibly dilutive shares to get the amount of shares you should add to your denominator **as long as the avg MP is higher than the exercise price, they are automatically going to have some diluted shares

what method do you use when you are trying to find dilutive EPS? you need to actually see if the warrants/options are possibly dilutive, so you use the _____________________ method.

The franchisee should capitalize the costs of acquiring the franchise. The capitalizable amount includes the initial fee and other expenditures (e.g., legal fees) necessary to acquire the franchise that will provide future benefits. If the initial fees are paid over a period longer than 1 year, the present value of the payments is capitalized as part of franchise costs and recognized as an intangible asset.

what should a franchisee do with the costs of acquiring a franchise?

asset

what sort of account is available-for-sale debt securities?

meter money finishing licenses hunting licenses

what sort of governmental money is accounted for under a cash basis?

other financing sources. you never have bonds payable or long term notes payable accounts.

what would credit in governmental accounting when you issue bonds?

The patent is recorded at the amount of the incidental costs, or $25,000 ($10,000 patent registration fees + $15,000 legal fees). The amounts paid for research and development must be expensed as incurred and are never capitalized as part of the cost of the asset.

when a company is developing an intangible asset (such as a patent), what costs do you expense and what to you assign as the asset's value to amortize?

When the fair value is less than cost, you recognize the loss. When the value is regained in the second year, the prior loss is undone with a gain of $10,000, but the additional $5,000 increase in value must go to an equity account in other comprehensive income.

when a company is reevaluating (fair value) an asset under IFRS, how do you account for a decrease or increase?

basic and dilutive EPS

when a corporation has a complex capital structure, what EPS calculations is it required to show?

executory costs such as insurance, mantenance, and taxes.

when calculating the minimum lease payment, what do you not include?

when they have a simple capital structure- no preferred dividends

when do you use basic earnings per share versus just earning per share?

when the parent controls the subsidiary- 50% or more ownership.

when do you use the acquisition method and prepare a consolidate financial statement?

interim financials. not appropriate for GAAP

when do you use the gross profit method?

when they spend it, not when they receive it (modified accrual)

when does a governmental unit record revenue from a grant?

on the income statement included in earnings.

when using the remeasurement method for foreign currency financial conversions, where does the remeasurement gain or loss belong?

Stockholder's equity- item of other comprehensive income

when using the translation method for foreign currency financial conversions, where does the translation gain or loss belong?

principle- face X *PV of $1* at the rate and amount of periods on interest- face X SR X *PV annuity*

when you are asked to find the amount you should issue the bond at, what PV table do you use for interest and what do you use for principle?

The ceiling is the inventory's NRV (NRV = sale price - selling cost) The floor is the NRV minus normal GP on sale

when you are using LIFO and valuing your inventory at lower of cost or market, market has a ceiling and a floor (limits it can no exceed or go below). What are these two amounts?

you reverse the original entry for the original amounts. 1. send out order DR encumbrance CR encumbrance outstanding 2. receive bill (reverse the original encumbrance entry) DR encumbrance outstanding CR encumbrances 3. record the actual bill DR expenditures CR vouchers payable

when you send out a purchase order under modified accrual accounting for governmental accounting, you make a journal entry because you have to keep track on encumbrance's (potential obligations). What do you do when you receive the actual bill?

debit loss from inventory write-down credit inventory

when you're using FIFO and measuring inventory at the lower of cost or NRV, what sort of entry do you make NRV is lower than cost?

since they have to treat purchases of fixed assets as expenditures, they report the asset on a statement called *government-wide statement of net position*. the government-wide statements use normal accrual accounting

where does a governmental fund record their fixed assets?

An impairment loss is recognized in income from continuing operations. An impairment cannot be reversed under GAAP- only IFRS

where on the financial statements is an impairment loss recorded?

translation method

which foreign currency method do you use to convert financials from foreign currency to US Dollars when the sub has most transactions done in their currency, they get their financing from banks in their currency, and there are very few transactions with the US parents...

Re-measurement method

which foreign currency method do you use to convert financials from foreign currency to US Dollars when the sub mostly functions in US dollars? Most transactions are done in US dollars, they get their financing from US banks, and a lot of transactions with US parents...

1, 3, and 4

which one do you back out salvage value? 1. straight line 2. double-declining method 3. sum of the years digits 4. units of production

GAAP

who prohibits reporting cash flow per share- IFRS or GAAP?

The most recent prices are on the balance sheet. You're shoving the old prices on the income statement for sales of inventory and leaving the most recent items in inventory on the balance sheet at the end of the year.

why id FIFO called the balance sheet approach?

The most recent prices are on the income statement.

why is LIFO called the income statement approach?

Answer (A) is correct. Plack Co. owns 2% of the stock of Ty Corp. Accordingly, this investment should be accounted for using the fair value method. If the fair value of the stock is not readily determinable, the measurement alternative may be selected. This alternative is cost minus any impairment, plus or minus changes resulting from observable price changes for the identical or a similar investment of the same issuer. Under either method, dividends from an investee are accounted for by the investor as dividend income unless a liquidating dividend is received. The recipient of a stock dividend does not recognize income. Thus, Plack should report dividend income of $24,000 [(10,000 shares + 2,000 shares received as a stock dividend on April 30) × $2 per share dividend].

Plack Co. purchased 10,000 shares (2% owner ship) of Ty Corp. on February 14 and did not elect the fair value option. Plack received a stock dividend of 2,000 shares on April 30, when the market value per share was $35. Ty paid a cash dividend of $2 per share on December 15. In its income statement for the year, what amount should Plack report as dividend income? A. $24,000 B. $90,000 C. $94,000 D. $20,000

Answer (C) is correct. The FVM is based on the highest and best use (HBU) by market participants. This use maximizes the value of the asset. The HBU is in-use if the value-maximizing use is in combination with other assets in a group. An example is machinery. The HBU is in-exchange if the value-maximizing use is as a standalone asset. An example is a financial asset.

The fair value measurement (FVM) of an asset A. Assumes transfer, not a settlement. B. Is based on the expected use by the reporting entity. C. Reflects the highest and best use by market participants. D. Includes the entity's own credit risk.

After results of continuing operations but before results of discontinued operations. Not reported net of tax unlike other comprehensive income

Where is a gain or loss from a transaction that is unusual in nature or infrequent in occurrence reported on the income statement?

Accounts receivable; Notes receivable

_____________________ often are short-term, unsecured, and informal credit arrangements (open accounts). _________________________ are evidenced by a formal instrument, such as a promissory note

FOB destination

___________________________ means control over goods passes to the buyer when the seller makes a proper tender of delivery of the goods at the destination. The seller should include the goods in inventory until that time.

lower of (historical) cost or NRV

at what cost do you value FIFO inventory at

Answer (C) is correct. Revenues are inflows or other enhancements of assets or settlements of liabilities from activities that constitute the entity's ongoing major or central operations. Thus, a revenue may result from a decrease in a liability from primary operations, for example, by delivering goods that were paid for in advance.

According to the FASB's conceptual framework, an entity's revenue may result from a(n) A. Decrease in an asset from primary operations. B. Increase in an asset from incidental transactions. C. Decrease in a liability from primary operations. D. Increase in a liability from incidental transactions.

Answer (A) is correct. Recognition is the process of formally recording or incorporating an item in the financial statements as an asset, liability, revenue, expense, gain, or loss.

According to the FASB's conceptual framework, the process of reporting an item in the financial statements of an entity is A. Recognition. B. Matching. C. Allocation. D. Realization.

Answer (B) is correct. "Fair value is the price that would be received to sell an asset or paid to transfer a liability in an orderly transaction between market participants at the measurement date." Thus, fair value is an exit price.

According to the FASB, fair value is A. An entity-specific measurement. B. An exit price. C. Based on an actual transaction. D. An entry price.

decreases; decreases

As the discount rate increases, the present value____________. Also, as the discount period increases, the present value ________________.

No

Do you include dividends in arrears in current liabilities?

No

Do you include dividends not declared in current liabilities?

C. Results in recognition of unrealized gains and losses in earnings of a business entity.

Election of the fair value option (FVO) for financial assets A. Requires deferral of related upfront costs. B. Permits only for-profit entities to measure eligible items at fair value. C. Results in recognition of unrealized gains and losses in earnings of a business entity. D. Results in recognition of unrealized gains and losses in other comprehensive income of a business entity.

Answer (C) is correct. Present value is in theory the most relevant method of measurement because it incorporates time value of money concepts. In practice, it is used only for noncurrent receivables and payables. Determination of the present value of an asset or liability requires discounting at an appropriate interest rate the related future cash flows expected to occur in the due course of business.

According to the FASB's conceptual framework, noncurrent payables are usually measured and reported at A. Current market value. B. Settlement value. C. Present value of future cash flows. D. Historical proceeds.

Answer (C) is correct. Relevance and faithful representation are the fundamental qualities that make accounting information useful for decision making. Relevance is the capacity of information to make a difference in the user's decision. A representation is perfectly faithful if it is complete, neutral, and free from error.

According to the FASB's conceptual framework, the two fundamental qualitative characteristics that make accounting information useful for decision making are A. Neutrality and completeness. B. Consistency and comparability. C. Relevance and faithful representation. D. Fairness and precision.

Answer (C) is correct. Cost is a pervasive constraint on the information provided by financial reporting. The benefits of financial information should exceed the costs of reporting. D.

According to the FASB's conceptual framework, the usefulness of providing information in financial statements is subject to the constraint of A. Consistency. B. Relevance. C. Cost. D. Representational faithfulness.

Answer (A) is correct. Several elements describe transactions, events, and circumstances during intervals of time, including investments by owners, distributions to owners, comprehensive income, revenues, expenses, gains, and losses. Rational allocation procedures are used in accrual accounting.

According to the FASB's conceptual framework, which of the following is not an element describing transactions, events, and circumstances during intervals of time? A. Rational allocation procedures. B. Distributions to owners. C. Comprehensive income. D. Investments by owners.

Answer (C) is correct. Internally generated cash flow projections are not observable and would be considered a Level 3 input. Level 3 inputs are unobservable inputs that are used in the absence of observable inputs. They should be based on the best available information in the circumstances.

Each of the following would be considered a Level 2 observable input that could be used to determine an asset or liability's fair value except A. Quoted prices for identical assets and liabilities in markets that are not active. B. Interest rates that are observable at commonly quoted intervals. C. Internally generated cash flow projections for a related asset or liability. D. Quoted prices for similar assets and liabilities in markets that are active

Answer (A) is correct. When an account receivable is written off, both accounts receivable and the allowance for uncollectible accounts are decreased. When an account previously written off is collected, the account must be reinstated by increasing both accounts receivable and the allowance. Accounts receivable is then decreased by the amount of cash collected.

When the allowance method of recognizing uncollectible accounts is used, the entries at the time of collection of a small account previously written off A. Increase the allowance for uncollectible accounts. B. Decrease the allowance for uncollectible accounts. C. Increase net income. D. Have no effect on the allowance for uncollectible accounts. Answer (D) is incorrect. The allowance is increased.

Credit risk

_______________________ is the risk of accounting loss from a financial instrument because of the possible failure of another party to perform. An entity must disclose most significant concentrations of credit risk arising from instruments. Group concentrations arise when multiple counterparties have similar characteristics that cause their ability to meet obligations to be similarly affected by change in conditions. An example of such a group is an industry.

FOB shipping point means control over goods passes to the buyer when the seller makes a proper tender of delivery of the goods to the carri

___________________________ means control over goods passes to the buyer when the seller makes a proper tender of delivery of the goods to the carrier. The buyer then includes the goods in inventory.

Answer (D) is correct. The balance sheet includes information that is often used in assessing liquidity and financial flexibility but should be used at minimum with a cash flow statement. Liquidity reflects nearness to cash. Financial flexibility is the ability to take action to alter cash flows so that the entity can respond to unexpected events.

In analyzing a company's financial statements, which financial statement will a potential investor primarily use to assess the company's liquidity and financial flexibility? A. Statement of cash flows. B. Statement of retained earnings. C. Income statement. D. Balance sheet.

FASB

The SEC has authority to regulate external financial reporting. It has chosen to allow the accounting profession (through the _____________) to establish principles.

The FASB's Accounting Standards Codification.

The SEC has the legal authority to establish financial reporting requirements and they have delegated some of this authority to the FASB. The SEC's authoritative accounting guidance is called the SEC pronouncements and the FASB has ______________.

Answer (D) is correct. The sale of a major division in the subsequent events period provides evidence of conditions not existing at the balance sheet date and thus does not require adjustment of the statements. Disclosure must be made, however, because the event is of such a nature that nondisclosure causes the statements not to be fairly presented. The form of the disclosure depends upon the significance of the event. In this case, many accounts are affected, and pro forma financial statements are the best method.

The entity's manufacturing division, whose assets constituted 75% of its total assets at September 30, Year 5 (end of year), was sold on November 1, Year 5. The new owner assumed the bonded indebtedness associated with this property. How should this event be presented in the financial statements? A. Disclosure in a note to the financial statements. B. No financial statement disclosure. C. Adjustment of the financial statements for the year ended September 30, Year 5. D. Disclosure by means of supplemental, pro forma financial data.

D.

The reporting entity may elect the fair value option (FVO) for A. An interest in a variable interest entity (VIE) if the reporting entity is the primary beneficiary. B. Its obligation for pension and other postretirement employee benefits. C. An investment consisting of more than 50% of the outstanding voting interests of another entity. D. Most financial assets and liabilities.

Answer (A) is correct. The Governmental Accounting Standards Board (GASB) was established by the Financial Accounting Foundation. It is the source of GAAP for state and local governments and is a private, nongovernmental organization.

The source of generally accepted accounting principles used by state and local governments in the United States is the A. Governmental Accounting Standards Board, a private, nongovernmental organization. B. Financial Accounting Standards Board, a private, nongovernmental organization. C. Federal Accounting Standards Advisory Board. D. U.S. Securities and Exchange Commission, a governmental organization.

The requirements for filing interim financial statements and pro forma information

Regulation S-X disclosure requirements and the SEC apply to

Answer (D) is correct. Accounting Standards Updates are issued to amend the Accounting Standards Codification (ASC). The ASC and SEC pronouncements are the only sources of authoritative financial accounting guidelines for nongovernmental entities in the U.S.

The FASB amends the Accounting Standards Codification through the issuance of A. Technical Bulletins. B. Staff Accounting Bulletins. C. Statements of Financial Accounting Standards. D. Accounting Standards Updates.

only at the end of the period after the physical count. The perpetual system debits COGS and credit inventory when a sale occurs so that they amount of inventory is constantly updated and correct.

The perpetual and periodic systems of recording inventory have the same result. However, under the periodic system, the amounts of inventory and cost of goods sold are updated

Answer (A) is correct. Stakeholders are individuals or groups that can affect or be affected by the organization's actions, objectives, or policies. They include managers, constituents, oversight bodies, and resource providers. B.

The stakeholders of NFPs include A. Resource providers. B. Shareholders. C. Members to whom certain economic benefits are provided directly. D. Investors.

Short-term, highly liquid investment like treasury bills, money market funds, and commercial paper. Also an investment with a maturity date in three months or less (interest rate is insignificant).

What is a cash equivalent

Answer (B) is correct. When an account receivable is written off, both accounts receivable and the allowance for uncollectible accounts are decreased. The entry is to debit the allowance and credit the receivable.

When the allowance method of recognizing bad debt expense is used, the allowance would decrease when a(n) A. Provision for uncollectible accounts is recorded. B. Specific uncollectible account is written off. C. Account previously written off is collected. D. Account previously written off becomes collectible.

excluded from earnings and reported in other comprehensive income (OCI).

Where do you report the temporary change in fair value for available-for-sale debt securities?

Answer (B) is correct. An entity recognizes in the financial statements adjusting events after the reporting period. These provide evidence about conditions existing at the end of the reporting period. However, the business combination did not occur until after year end. Hence, it required only disclosure, not recognition in the statements.

Which of the following material events occurring after the December 31, Year 6, end of the reporting period does not ordinarily result in adjustment of the financial statements before they are issued on February 28, Year 7? A. A 3-for-5 reverse stock split consummated on January 20, Year 7. B. A major business combination completed on January 20, Year 7. Negotiations had begun in December of Year 6. C. Write-off of a receivable from a debtor who had suffered from a deteriorating financial condition for the past 6 years. The debtor filed for bankruptcy on January 20, Year 7. D. Settlement of extended litigation on January 20, Year 7, in excess of the recorded year-end liability

3 is correct. Providing information for assessing service efforts and accomplishments is one of the objectives that serves to assist in evaluating the operating results of a governmental entity.

Which of the following objectives of financial reporting is applicable to governmental entities? Provide information 1. Useful in assessing management stewardship and performance. 2. About economic resources, obligations, net resources, and changes in them. 3. For assessing service efforts and accomplishments. 4. Useful in making resource allocation decisions.

Answer (A) is correct. Level 1 inputs are the most reliable. They are unadjusted quoted prices in active markets for identical assets or liabilities that the entity can access at the measurement date.

Which of the following phrases best describes a Level 1 input for measuring the fair value of an asset or liability? A. Unadjusted quoted prices for identical assets or liabilities in active markets. B. Inputs that are principally derived from or corroborated by observable market data. C. Quoted prices for similar assets or liabilities in active markets. D. Inputs for the asset or liability based on the reporting entity's internal data.

Answer (A) is correct. The correction of an error occurring in a prior period should be accounted for as a prior-period adjustment. It should be charged or credited net of tax to retained earnings and reported as an adjustment in the statement of equity. It is not included in net income for the current period.

Which of the following should be reflected, net of applicable income taxes, in the statement of equity as an adjustment of the opening balance in retained earnings? A. Correction of an error in previously issued financial statements. B. A material transaction that an entity considers to be unusual in nature. C. Cumulative effect of a change in depreciation method. D. Loss on disposal of a material component of an entity.

Answer (A) is correct. The statement of shareholders' equity (changes in equity) presents a reconciliation in columnar format of the beginning and ending balances in the various shareholders' equity accounts. A statement of changes in equity may include, for example, columns for (1) totals, (2) comprehensive income, (3) retained earnings, (4) accumulated OCI (but the components of OCI are presented in another statement), (5) common stock, and (6) additional paid-in capital.

The statement of shareholders' equity shows a A. Reconciliation of the beginning and ending balances in shareholders' equity accounts. B. Reconciliation of net income to net operating cash flow. C. Computation of the number of shares outstanding used for earnings per share calculations. D. Listing of all shareholders' equity accounts and their corresponding dollar amounts.

Answer (B) is correct. Assets are normally listed in the order of their importance, with current assets typically being the most important. For a public utility, the physical plant is the most important asset. Thus, public utilities often report their noncurrent assets as the first item on the balance sheet. This departure from the customary presentation in accordance with GAAP is justified by the unique operating characteristics of the industry.

A Midwestern public utility reports noncurrent assets as the first item on its statement of financial position. This practice is an example of the A. Going-concern assumption. B. Industry practice constraint. C. Conservatism constraint. D. Economic-entity assumption.

Total other comprehensive income is transferred to a component of equity separate from retained earnings and additional paid-in capital.

where is accumulated comprehensive income placed on the financials?

1. Understandability 2. Reliability 3. Relevance 4. Timeliness 5. Comparability 6. Cost-benefit limitations

Qualitative characteristics of state and local governmental accounting

0-19% which is little or no influence - FVM 20-49% which is significant influence - equity method or FVO 50-100% which is control - consolitaion

What are the three categories of an equity security (investment in voting stock)?

noncurrent receivables and payables.

Present value is in theory the most relevant method of measurement because it incorporates time value of money concepts. In practice, it is used only for ___________________.

Expenses that are incurred for the direction of the unity as a whole. They do jot relate entirely to a specific function. They include -legal -professional services -officers' salaries -utilities

What are general and administrative expenses?

Must be reported net of tax. Includes unrealized holding gains and losses. gains and losses on derivatives. Pension plan information. Foreign items. Reported below net income.

Other Comprehensive Income (OCI)

An outstanding check is a check that has been written by a company (and deducted from the appropriate general ledger cash account) but it has not yet cleared the bank account on which it is drawn.

Outstanding Check

Relevance a. Predictive value b. Confirmatory value c. Materiality (relevance predicts or confirms materiality) Faithful representation a. Completeness b. Neutrality c. Freedom from error (faithful representation is complete, neutral, and free from error)

Qualitative characteristics of useful financial- Relevance and Faithful representation apects

Answer (D) is correct. Accounting Standards Updates are issued to amend the Accounting Standards Codification (ASC). The ASC and SEC pronouncements are the only sources of authoritative financial accounting guidelines for nongovernmental entities in the U.S.

The FASB amends the Accounting Standards Codification through the issuance of A. Statements of Financial Accounting Standards. B. Technical Bulletins. C. Staff Accounting Bulletins. D. Accounting Standards Updates.

Answer (B) is correct. Accrual-basis amounts used in financial reporting are not useful to managers making day-to-day operating decisions. The practice of management accounting fulfills the needs of these users.

The financial statements included in the annual report to the shareholders are least useful to which one of the following? A. Competing businesses. B. Managers in charge of operating activities. C. Stockbrokers. D. Bankers preparing to lend money.

Noncurrent asset. Cannot be classified as cash or cash equivalent.

How do you classify cash in bond sinking fund account

The average time betweeb the acquisition of resources and the final receipt of cash from their sale. If the cycle is less than one year, one year is the period used for segregating currebt from noncurrent assets.

How long is the operating cycle? What does the operating cycle help define on the balance sheet?

Answer (B) is correct. Historical cost is the amount of cash, or its equivalent, paid to acquire an asset. Thus, the LCM rule departs from the historical cost principle when the utility of the inventory is judged no longer to be as great as its cost.

Reporting LIFO inventory at the lower of cost or market (LCM) is a departure from the accounting principle of A. Full disclosure. B. Historical cost. C. Consistency. D. Conservatism.

Answer (A) is correct. The financial capital maintenance approach requires that comprehensive income be determined by finding the change in equity (net assets) after adjusting for investments by, and distributions to, owners. However, this approach does not provide the detail of the transaction approach to income determination under which each component of income is measured and reported. Change in net assets $ (5,000) Capital stock sold (8,000) Dividends declared 3,000 Total $(10,000)

A company has beginning net assets of $100,000 and ending net assets of $95,000. During the year, additional capital stock was sold for $8,000, and dividends of $3,000 were declared. Using the capital maintenance approach, the net income (loss) for the year is calculated as A. $(10,000) B. $0 C. $(5,000) D. $5,000

Answer (D) is correct. Changes in estimated cash flows may result in (1) a fresh-start measurement or (2) a change in the plan of interest amortization. Given no remeasurement, the interest amortization plan may be revised using a catch-up method. Adjusting the carrying amount to the present value of the remaining cash flows discounted at the original rate is preferred by the FASB.

Changes in the estimated cash flows used in a present value measurement may be addressed by revising the interest amortization plan. This method is the alternative to a fresh-start remeasurement. The FASB's preferred expected cash flow approach to revision of the plan is to A. Calculate present value using one set of estimated cash flows. B. Determine prospectively a new effective rate. C. Determine retrospectively a new effective rate. D. Discount remaining cash flows at the original rate.

Answer (D) is correct. Under the fair value option, dividends received and unrealized gains and losses on remeasurement of financial assets to fair value are reported in earnings. Thus, the $1,800 of dividend income received and the $3,000 ($35,000 - $32,000) of unrealized loss are reflected in Gilman's earnings for Year 3. This results in a total loss of $1,200 ($1,800 - $3,000) attributable to the Meteor stock investment.

During Year 3, Gilman Co. purchased 5,000 shares of the 500,000 outstanding shares of Meteor Corp.'s common stock for $35,000. During Year 3, Gilman received $1,800 of dividends from its investment in Meteor's stock. The fair value of Gilman's investment on December 31, Year 3, is $32,000. Gilman has elected the fair value option for this investment. What amount of income or loss that is attributable to the Meteor stock investment should be reflected in Gilman's earnings for Year 3? A. Income of $1,800. B. Loss of $3,000. C. Income of $4,800. D. Loss of $1,200.

Answer (B) is correct. Under U.S. GAAP, a current liability is classified as noncurrent if the entity (1) intends to refinance on a noncurrent basis and (2) demonstrates an ability to complete such refinancing. The ability to refinance may be demonstrated by entering into an agreement to refinance or to reschedule payments on a long-term basis. Such an agreement may be completed after the reporting date but before the financial statements are issued. Under IFRS, a current liability may be classified as noncurrent only if the agreement to refinance or reschedule payments on a noncurrent basis is completed before the reporting date.

A company had $100,000 in current liabilities at the end of the current year. The company refinanced this liability on a noncurrent basis subsequent to the end of the year but before the financial statements were issued. How should this liability be presented, according to IFRS and U.S. GAAP, in the company's year-end financial statements? A. In noncurrent liabilities under IFRS and in current liabilities under U.S. GAAP. B. In current liabilities under IFRS and in noncurrent liabilities under U.S. GAAP. C. In current liabilities under IFRS and U.S. GAAP. D. In noncurrent liabilities under IFRS and U.S. GAAP.

Answer (A) is correct. The content of the MD&A section is determined by regulations of the SEC. The MD&A, standard financial statements, summarized financial data for at least 5 years, and other matters must be included in annual reports to shareholders and in Form 10-K filed with the SEC. Forward-looking information in the form of forecasts is encouraged in the MD&A but not required.

The content of the Management's Discussion and Analysis (MD&A) section of an annual report is A. Mandated by regulations of the Securities and Exchange Commission. B. Mandated by pronouncements of the Financial Accounting Standards Board. C. Reviewed by independent auditors. D. Mandated by regulations of the Internal Revenue Service.

Answer (B) is correct. Fiscal accountability is the responsibility of governments to justify that their actions currently comply with public decisions concerning the raising and spending of public resources in the short term. Operational accountability is a government's responsibility to report the extent to which it has met accounting objectives efficiently and effectively, using all resources available, and whether it can continue to do so in the near future. The governmental funds financial statements focus on the fiscal accountability of governmental activities. However, government-wide financial statements focus on the operational accountability of the governmental and business-type activities of the government as a whole. The financial statements of fiduciary funds and proprietary funds provide information about operational accountability.

The statement of activities of the government-wide financial statements is designed primarily to provide information to assess which of the following? A. Financial accountability. B. Operational accountability. C. Functional accountability. D. Fiscal accountability.

The entry to record bad debt expense under the allowance method is to debit bad debt expense and credit the allowance account. When a specific account is then written off, the allowance is debited and accounts receivable credited. Net income is affected when bad debt expense is recognized, not at the time of the write-off. Because accounts receivable and the allowance account are decreased by the same amount, a write-off of an account also has no effect on the net amount of accounts receivable.

Under the allowance method of recognizing uncollectible accounts, the entry to write-off an uncollectible account A. Has no effect on the allowance for uncollectible accounts. B. Decreases net income. C. Has no effect on net income. D. Increases the allowance for uncollectible accounts.

4. is correct. A basic feature of financial accounting is that the business entity is assumed to be a going concern in the absence of evidence to the contrary. The going-concern concept is based on the empirical observation that many entities have indefinite lives. The reporting entity is assumed to have a life long enough to fulfill its objectives and commitments and therefore to depreciate wasting assets over their useful lives.

A newly acquired plant asset is to be depreciated over its useful life. What is the basis for this accounting method? 1. Materiality 2. Economic-entity assumption 3. Monetary-unit assumption 4. Going-concern assumption

Answer (B) is correct. If 10% or more of revenue is derived from sales to any single customer, the entity must disclose that information and the amount of revenue from each such customer (without disclosing the identity of the customer). The identity of the operating segment or segments making the sales must also be disclosed. Hence, sales to a single customer of $5,000,000 ($50,000,000 total revenue × 10%) will necessitate disclosure of major customer data.

Grum Corp. is a publicly owned corporation subject to the requirements for segment reporting. In its income statement for the year ended December 31, Grum reported revenues of $50 million, operating expenses of $47 million, and net income of $3 million. Operating expenses include payroll costs of $15 million. Grum's combined assets of all operating segments at December 31 were $40 million. In its financial statements for the current year, Grum should disclose major customer data if sales to any single customer amount to at least A. $1,500,000 B. $5,000,000 C. $300,000 D. $4,000,000

Answer (A) is correct. Material unusual or infrequent items, and gains or losses from disposal of a component of an entity are (1) separately disclosed in the interim statements, (2) included in interim-period net income, and (3) not prorated over the year.

How are discontinued operations and material unusual or infrequently occurring items that occur at midyear initially reported? A. Included in net income and disclosed in the notes to interim financial statements. B. Disclosed only in the notes to interim financial statements. C. Disclosed only in the notes to the year-end financial statements. D. Included in net income and disclosed in the notes to the year-end financial statements.

Answer (D) is correct. Unearned revenue is a liability arising from collections in advance of delivering goods or performing services. Current liabilities generally are expected to be settled or liquidated in the ordinary course of business during the longer of 1 year or the operating cycle. Thus, rental income paid for 8 months in advance by the tenants is reported as a current liability in the financial statements.

How should unearned rent that has already been paid by tenants for the next eight months of occupancy be reported in a landlord's financial statements? A. Long-term liability. B. Current asset. C. Long-term asset. D. Current liability.

The revenue recognized must reflect the price that a customer would have paid for the promised goods or services if the cash payment had been made when the goods were transferred to the customer (the cash selling price). Because the customer obtained control over the machine at contract inception, the revenue from this contract of $250,000 was recognized on 1/1/Year 1. Thus, interest income from the adjustment of the transaction price for the effect of the time value of money must be recognized using the effective interest method. The interest component of the first installment payment on 12/31/Year 1 is $25,000 ($250,000 × 10%). The remaining amount of the principal to be paid is $130,951 [$250,000 - ($144,049 annual payment - $25,000 Year 1 interest)]. Interest income for Year 2 is therefore $13,095 ($130,951 × 10%). D.

On January 1, Year 1, Sam Co. entered into a contract with a customer to sell a machine for two annual payments of $144,049 starting at the end of Year 1. The customer obtains control of the machine at contract inception. The cash selling price of the machine is $250,000. Sam determined that (1) the contract includes a significant financing component and (2) the contract includes an implicit interest rate of 10% what revenue does Sam Co. recognize and when?

Answer (D) is correct. If a performance obligation is not satisfied over time, an entity satisfies the performance obligation at a point in time. The performance obligation is satisfied and revenue is recognized when the customer obtains control of a promised asset. The indicators of the transfer of control that should be considered include (1) the entity's present right to payment for the asset, (2) the customer's legal title to the asset, (3) the entity's transfer of physical possession of the asset, (4) the customer's significant risks and rewards of ownership of the asset, and (5) the customer's acceptance of the asset.

Robin Gavaskar, who recently founded a company that produces baseball bats and balls, wants to determine her company's policy for revenue recognition. The most appropriate time to recognize revenue for the goods is when A. Cash is received. B. Production is completed. C. Quarterly financial statements are prepared. D. The entity has transferred physical possession.

Answer (B) is correct. In Year 2, a $400,000 contingent loss and an accrued liability in the amount of $400,000 were properly recognized. In Year 3, the actual loss of $310,000 ($250,000 cash + $60,000 carrying amount of the copyright) was $90,000 less than the previously estimated amount. This new information should be treated as a change in estimate and accounted for in the period of change. Consequently, the $90,000 difference will be credited to Year 3 income as a recovery of a previously recognized loss.

Tone Company is the defendant in a lawsuit filed by Witt in Year 2 disputing the validity of a copyright held by Tone. At December 31, Year 2, Tone determined that Witt would probably be successful against Tone for an estimated amount of $400,000. Appropriately, a $400,000 loss was accrued by a charge to income for the year ended December 31, Year 2. On December 15, Year 3, Tone and Witt agreed to a settlement providing for a cash payment of $250,000 by Tone to Witt, and the transfer of Tone's copyright to Witt. The carrying amount of the copyright on Tone's accounting records was $60,000 at December 15, Year 3. What would be the effect of the settlement on Tone's income before income tax in Year 3? A. $150,000 increase. B. $90,000 increase. C. $60,000 decrease. D. No effect.

Answer (A) is correct. Business-type activities often perform only a single function. Thus, business-type activities of different governments are more easily compared than governmental-type activities.

Which of the following does not describe a difference between the business-type activities and the governmental-type activities of a governmental entity? A. Business-type activities often perform multiple functions and are thus more easily compared between governments than are governmental-type activities. B. Business-type activities have large investments in revenue-producing capital assets. C. Business-type activities involve a direct exchange of money in return for goods delivered or services rendered. D. Business-type activities adopt budgets, but they often lack the legal force of the budget for governmental-type activities.

Answer (A) is correct. One set of disclosures concerns risks and uncertainties relating to the nature of operations. Thus, entities must disclose their (1) major products or services, (2) principal markets, and (3) the locations of those markets. The company must disclose its major products. They also should disclose (1) all industries in which they operate; (2) the relative importance of each; and (3) the basis for determining the relative importance. Thus, the company must disclose the industries in which it operates.

Which of the following information is required to be disclosed in the notes to the financial statements? A. The company operates in both the soft drink industry and alcohol industry. B. It is likely that the company will face decreasing revenue and increasing costs within one year. C. The company's intent to acquire one of its main competitors. D. Cash flow that was used in financing and investing activities of the compan.

Answer (D) is correct. The operating environments of NFPs and business entities are similar in many ways. Both produce and distribute goods and services using scarce resources.

Which of the following is a characteristic of nongovernmental not-for-profit entities (NFPs)? A. Business entities and NFPs usually obtain resources in the same way. B. Noneconomic reasons seldom underlie the decision to provide resources to NFPs. C. The operating environment of NFPs ordinarily differs from that of business entities. D. Both NFPs and business entities use scarce resources in the production and distribution of goods and services.


Kaugnay na mga set ng pag-aaral

ELNEC Pallative Care Nursing. End of LIfe Nursing Education Consortium. Module 2: Communication in Palliative Care. Section 2: Communication Techniques

View Set

LL Practice Quiz (150 questions)

View Set

A&P: Chapter 11 Practice Questions

View Set